Re: [obm-l] Fwd: Identidade de Euler

2011-05-01 Por tôpico Eder Albuquerque
Pensei que não se pudesse fazer propaganda aqui...

Em 29 de abril de 2011 10:34, Luís Lopes qed_te...@hotmail.com escreveu:

  Sauda,c~oes,

 Este é o exercício 14 do Manual de Seq. e Séries Volume 2.
 A resolução lá apresentada é outra: identidade de polinômios
 (uma outra técnica básica e útil para resolver identidades deste tipo).

 A página http://www.escolademestres.com/
 contém uma amostra dele.

 []'s
 Luís



 --
 Date: Thu, 28 Apr 2011 17:40:10 -0300
 Subject: Re: [obm-l] Fwd: Identidade de Euler
 From: victorcar...@globo.com
 To: obm-l@mat.puc-rio.br


 Oi  Mestre  Nehab ,
 Gostei da sugestão e mais ainda  das n pessoas que moram em Nilópolis (
 minha  terrinha).

 Abraços

 Carlos  Victor

 Em 28 de abril de 2011 17:21, Carlos Nehab ne...@infolink.com.brescreveu:

 Oi, Fábio,

 Não resisti:

 Resolva os seguinte problema de duas maneiras (uma técnica básica e útil
 para resolver identidades deste tipo).
 De quantas maneira posso formar comissões de p pessoas, a partir de um
 total de m + n pessoas, sendo m o total de pessoas que moram no Maracanã e n
 as pessoas que moram em Nilópolis?

 Abraços,
 Nehab

 Em 28/4/2011 13:24, fabio henrique teixeira de souza escreveu:

 -- Mensagem encaminhada --
 De: fabio henrique teixeira de souzafabiodja...@ig.com.br
 Data: 28 de abril de 2011 08:52
 Assunto: Identidade de Euler
 Para: obm-l@mat.puc-rio.br


 Pessoal, estou batendo cabeça e não consigo demonstrar que
 C(m,0).C(n,p) + C(m,1).C(n,p-1) + C(m,2).C(n,p-2) + ... + C(m,p).C(n,0) =
 C(m+n,p)

 Alguém pode me dar uma dica?


 =
 Instruções para entrar na lista, sair da lista e usar a lista em
 http://www.mat.puc-rio.br/~obmlistas/obm-l.html
 =





[obm-l] outra soma de série

2011-01-20 Por tôpico Eder Albuquerque
Pessoal,
 
Como calcular a soma de 1/[n(n+1)(n+2)...(n+p)], com n de 1 a infinito, e p 
natural fixado?
 
Já tentei usar frações parciais, porém não consegui muita coisa...
 
Obrigado,
 
Eder


  

[obm-l] soma de série

2011-01-17 Por tôpico Eder Albuquerque
Olá a todos.
Alguém tem uma dica para calcular o somatório de 1/[(4n+1)(4n+3)] com n 
variando de 1 a infinito?
Obrigado,
Eder


  

[obm-l] Re: [obm-l] Re: [obm-l] Re: [obm-l] construir bijeção

2011-01-15 Por tôpico Eder Albuquerque
Pessoal, a idéia é bem interessante. Obrigado mesmo.
Eder

--- Em sáb, 15/1/11, Bernardo Freitas Paulo da Costa bernardo...@gmail.com 
escreveu:

De: Bernardo Freitas Paulo da Costa bernardo...@gmail.com
Assunto: [obm-l] Re: [obm-l] Re: [obm-l] construir bijeção
Para: obm-l@mat.puc-rio.br
Data: Sábado, 15 de Janeiro de 2011, 6:06

2011/1/15 Pedro Angelo pedro.fon...@gmail.com:
 eu tive a mesma dúvida um tempo atrás, e achei esse artigo aqui
 http://planetmath.org/encyclopedia/ClosedOpen.html , que eu achei muito bom.
 Ele dá duas demonstrações de que os dois conjuntos (o aberto e o fechado)
 têm a mesma cardinalidade. A primeira delas é o seguinte existe uma
 injetiva de um no outro, e uma injeiva do outro no um, portanto pelo teorema
 de Zwardjenjizfgyulpoz
= Cantor-Bernstein-Schroder. Coitados deles, não mutile o nome assim
;-) (Ah, e o tal do teorema não é tão impossível assim de ser
demonstrado não!! Não se deixem assustar, a idéia é muito bonita, e,
de certa forma, intuitiva: vale a pena ver e entender)

 existe uma bijeção entre os dois. Na segunda
 demonstração, ele de fato constrói uma bijeção que é, em suma, o seguinte:
 ele enumera o conjunto dos racionais entre 0 e 1, de modo que os dois
 primeiros racionais sejam o próprio zero e o um. Aí ele faz uma bijeção
 entre os dois conjuntos arrastando o conjunto dos racionais duas unidades
 pra direita (ou seja, o primeiro racional vira o terceiro, o segundo
 vira o quarto, o terceiro vira o quinto, etc.), e deixando invarianes os
 irracionais.

 eu levei um tempo até acreditar e entender. Quando eu tava pesquisando sobre
 isso, eu tinha na cabeça a idéia de que essa bijeção tinha que ser contínua.
 Agora eu já acho que é meio óbvio que não dá pra uma bijeção entre esses
 dois conjuntos ser contínua. (essa em particular não é conínua em nenhum
 ponto!)
Talvez seja legal *provar* que não existe bijeção contínua entre [0,1]
e (0,1). Dica: considere f: [0,1] - (0,1) e em seguida f( (0,1] ).

Outra coisa é que a sua bijeção pode ser bem simplesmente modificada
(na verdade, segundo a idéia do Renji) : escolha em vez de todos os
racionais, apenas uma quantidade infinita deles (tente deixar a maior
parte de lado, mesmo que isso não faça muito sentido...). Você pode
pensar como aproximações decimais de uma dízima periódica, por
exemplo. Ao fazer isso, a função que você construir, com um shift
nesse conjunto, e identidade no resto, será contínua em todos os
pontos *exceto* no conjunto infinto que você escolheu (e no limite
dele também, claro). Uma pergunta talvez mais difícil seria: será que
é possível fazer uma bijeção de [0,1] em (0,1) que seja descontínua
num número *finito* de pontos ? Se sim, qual é o mínimo de
descontinuidades ?

 boa sorte : )

Abraços,
-- 
Bernardo Freitas Paulo da Costa

=
Instruções para entrar na lista, sair da lista e usar a lista em
http://www.mat.puc-rio.br/~obmlistas/obm-l.html
=



  

[obm-l] construir bijeção

2011-01-14 Por tôpico Eder Albuquerque
Olá, 
Alguém tem idéia de como construir uma bijeção  f : (0,1)  [0,1]? É 
possível?
Obrigado,
Eder 


  

[obm-l] indução finita

2011-01-09 Por tôpico Eder Albuquerque
Pessoal,
Depois de passar muito tempo meditando sobre o exercício abaixo (consta num 
artigo do Elon Lages Lima publicado na Eureka), resolvi enviar para a lista. Se 
alguém puder resolver, fico muito agradecido... Eis a questão:
Para todo n em N, ponha x_n = { (n+1)^2 / [n(n+2)] }^n e prove por indução que 
se tem x_n  (n+2)/(n+1). Conclua que a seqüência de termo geral x_n 
=[(n+1)/n]^n é crescente.
Sugestão: x_(n+1)=[(n+2)/(n+1)]^3.[n/(n+3)].x_n.   (será que está certo 
isso???).
Obrigado,
Eder


  

[obm-l] Equação

2008-12-21 Por tôpico Eder Albuquerque
Olá a todos,

Alguém poderia me confirmar se a equação abaixo tem mais de uma solução nos 
inteiros:

y^2 - 3 = x(3y - 6)


Cheguei facilmente a uma solução, mas não sei se pára aí.

Obrigado.



 


  Veja quais são os assuntos do momento no Yahoo! +Buscados
http://br.maisbuscados.yahoo.com

[obm-l] equação

2008-12-20 Por tôpico Eder Albuquerque
Olá a todos,

Alguém poderia me confirmar se a equação abaixo tem mais de uma solução nos 
inteiros:

y^2 - 3 = x(3y - 6)


Cheguei facilmente a uma solução, mas não sei se pára aí.

Obrigado.





  Veja quais são os assuntos do momento no Yahoo! +Buscados
http://br.maisbuscados.yahoo.com

[obm-l] Re: [obm-l] Re: [obm-l] perímetro mínimo

2008-06-25 Por tôpico Eder Albuquerque
Obrigado pelos esclarecimentos. Estou analisando aqui.

--- Em seg, 23/6/08, Luiz Alberto Duran Salomão [EMAIL PROTECTED] escreveu:

De: Luiz Alberto Duran Salomão [EMAIL PROTECTED]
Assunto: [obm-l] Re: [obm-l] perímetro mínimo
Para: obm-l@mat.puc-rio.br
Data: Segunda-feira, 23 de Junho de 2008, 10:37

Caro Ponce:

Creio que sua intenção foi dizer que C é a interseção de OY com BP (e
não AP) e que B estaria na interseção de OX com CQ (e não AQ),
não é mesmo?

Abraços,
Luiz Alberto


- Original Message - 
From: Rogerio Ponce [EMAIL PROTECTED]
To: obm-l@mat.puc-rio.br
Sent: Sunday, June 22, 2008 6:40 PM
Subject: Re: [obm-l] perímetro mínimo


 Ola'  Eder,
 suponhamos que o ponto B ja' estivesse marcado, e que estamos apenas
 procurando pelo ponto C otimo, sobre OY.

 Nesse caso, para minimizar BC + CA , vemos que C e' a intersecao de OY
 com AP , onde P e' o ponto simetrico de A em relacao a OY.

 O mesmo aconteceria se C ja' estivesse marcado, e estivessemos
 procurando pelo ponto B otimo, que estaria na intersecao de OX com AQ,
 onde Q e' o simetrico de A em relacao a OX.

 Portanto, como cada um dos vertices C e B necessariamente otimiza a
 soma de suas distancias aos outros dois vertices, basta localizar os
 simetricos de A em relacao a OX e OY, e uni-los, de forma a determinar
 os vertices C e B.

 []'s
 Rogerio Ponce


 Em 22/06/08, Eder Albuquerque[EMAIL PROTECTED] escreveu:
 Por gentileza, ajudem-me na questão abaixo


 Dado um ângulo agudo XOY e um ponto interior A, achar um ponto B
sobre OX 
 e
 um ponto C sobre OY tais que o perímetro do triângulo ABC seja
mínimo.


   Novos endereços, o Yahoo! que você conhece. Crie um email novo
com 
 a
 sua cara @ymail.com ou @rocketmail.com.
 http://br.new.mail.yahoo.com/addresses

 =
 Instruções para entrar na lista, sair da lista e usar a lista em
 http://www.mat.puc-rio.br/~obmlistas/obm-l.html
 =



 -- 
 No virus found in this incoming message.
 Checked by AVG.
 Version: 7.5.524 / Virus Database: 270.4.1/1514 - Release Date: 23/6/2008 
 07:17

 

=
Instruções para entrar na lista, sair da lista e usar a lista em
http://www.mat.puc-rio.br/~obmlistas/obm-l.html
=


  Novos endereços, o Yahoo! que você conhece. Crie um email novo com a sua 
cara @ymail.com ou @rocketmail.com.
http://br.new.mail.yahoo.com/addresses

[obm-l] perímetro mínimo

2008-06-22 Por tôpico Eder Albuquerque
Por gentileza, ajudem-me na questão abaixo
 
 
Dado um ângulo agudo XOY e um ponto interior A, achar um ponto B sobre OX e um 
ponto C sobre OY tais que o perímetro do triângulo ABC seja mínimo.


  Novos endereços, o Yahoo! que você conhece. Crie um email novo com a sua 
cara @ymail.com ou @rocketmail.com.
http://br.new.mail.yahoo.com/addresses

[obm-l] perímetro mínimo

2008-06-21 Por tôpico Eder Albuquerque
Por gentileza, ajudem-me na questão abaixo
 
 
Dado um ângulo agudo XOY e um ponto interior A, achar um ponto B sobre OX e um 
ponto C sobre OY tais que o perímetro do triângulo ABC seja mínimo.


  Novos endereços, o Yahoo! que você conhece. Crie um email novo com a sua 
cara @ymail.com ou @rocketmail.com.
http://br.new.mail.yahoo.com/addresses

[obm-l] mdc

2008-03-27 Por tôpico Eder Albuquerque
Pessoal, o problema a seguir caiu numa prova  de teoria dos números que fiz 
ontem e foi a única dúvida...
   
  Provar:
   
  mdc(a,b)= 1   =  mdc(a+b,a²-ab+b²) =1 ou 3
   
  Agradeço se alguém mostrar como se prova.
   
  Eder

   
-
Abra sua conta no Yahoo! Mail, o único sem limite de espaço para armazenamento! 

[obm-l] probabilidade 2

2006-03-21 Por tôpico Eder Albuquerque
Pessoal, aqui vai um problema que tô achando meio obscuro...Em um grupo de galinhas, existem algumas doentes. Sabe-se que 0,1% das galinhas estão doentes. Deseja-se fazer um teste para detectar se uma determinada galinha está doente. Sabe-se que a probabilidade do teste dar positivo ( indica que a galinha está doente ) estando uma galinha boa é de 1% e que a probabilidade do teste dar negativo estando a galinha doente é de 0,9%. Qual a probabilidade de a galinha estar realmente doente, sendo que foram realizados n testes? (p deram positivos e q = n - p deram negativos).
		 
Yahoo! Acesso Grátis  
Internet rápida e grátis. Instale o discador agora!

[obm-l] Probabilidade

2006-03-15 Por tôpico Eder Albuquerque
Pessoal, tô com dúvidas nesta:Suponha que n homens, numa festa, atirem seus paletós no guarda-roupas. Os paletós são misturados e cada um deles deverá selecionar aleatoriamente um paletó. Calcule a probabilidade de que ao menos um dos homens selecione o seu prórpio paletó.Se alguém puder me mostrar como faz, agradeço.Eder
		 
Yahoo! Acesso Grátis 
Internet rápida e grátis. Instale o discador agora!

[obm-l] funções

2005-10-20 Por tôpico Eder Albuquerque
Olá,

Pessoal, essa é velha, mas não tô lembrando como fazer... A questão é: mostre que toda função de variável real pode ser escrita como a soma de uma função real ímpar com uma função real par.

Obrigado pela ajuda,

Eder
		 
Promoção Yahoo! Acesso Grátis: a cada hora navegada você acumula cupons e concorre a mais de 500 prêmios! Participe!

[obm-l] geometria

2005-07-19 Por tôpico Eder Albuquerque
Olá,

Gostaria de ajuda no seguinte problema: seja ABC um triângulo isósceles, onde AB=AC são tangentes a uma circunferência e BC é uma corda. Seja P um ponto sobre a circunferência anterior, interno ao triângulo ABC, tal que a distância de P a AB é9 e a distância de P a AC é 4. Encontre a distância de P a BC.

Não tô conseguindo resolver...

Grato,

Eder__Converse com seus amigos em tempo real com o Yahoo! Messenger http://br.download.yahoo.com/messenger/ 

Re: [obm-l] Problema de trigonometria

2005-04-30 Por tôpico Eder Albuquerque
Olá Valdemir,

Respondi ao Ronaldo no email dele. Na verdade não sou adepto de frases, poemas etc, para lembrar fórmulas, como talvez possa estar parecendo. Segue o email que enviei para ele:


Oi Ronaldo,

Temendo que a msg ficasse muito off-topic, resolvi mandar para o seu email.

Para ser sincero, eu nunca fui muito chegado a métodos como poemas, músicas, frases etc. Acredito que a melhor forma de não esquecer essas relações ( e outras)é fazer muitos, mas muitos exercícios, em que elas são exigidas. Além disso, ver como são demonstradas ajuda...

Quando fazia cursinho para o ITA, resolvi mais de 700 problemas só de trigonometria (tive a curiosidade de contar) e, depois disso, não tinha como não fixar essas fórmulas. Tão importante quanto fixar, é ver como aplicá-las eficientemente nos problemas... Precisa muito "treino".

Felizmente, consegui entrar no ITA. Valeu a pena, hehehe.

Quanto à dedução, até que é fácil. Veja:

sen(a+b)=senacosb+senbcosa
sen(a-b)=senacosb-senbcosa

Somando:

sen(a+b)+sen(a-b)=2senacosb

faça a+b=p e a-b=q, resulta a=(p+q)/2 e b=(p-q)/2, donde

senp+senq=2sen[(p+q)/2]cos[(p-q)/2]

As outras relações vc obtém de forma análoga...

Falô,
Valdemir [EMAIL PROTECTED] wrote:




Olá Éder, é que eu me lembrei de uma outra para o cossenoda soma, suponha que vc queira calcular o cos(a+b), o poeminha é assim: "Coça A coça B, troca o sinal sem sabê"
Acho que é mais boba ainda que a do seno, mas eu nunca mais me esqueci.
Um abraço 
Dema.

- Original Message - 

From: Ronaldo Luiz Alonso 
To: obm-l@mat.puc-rio.br 
Sent: Saturday, April 30, 2005 4:04 AM
Subject: Re: [obm-l] Problema de trigonometria

Oi Éder como você consegui decorar todas essas fórmulas?
Tem algum truque? Tipo daqueles que usamos
para deocorar a tabela periódica?
O pessoal da lista quer saber ! :)

sena+senb= 2sen[(a+b) / 2] cos[(a-b)/ 2]
sena-senb= 2sen[(a-b)/2]cos[(a+b)/2]
cosa+cosb= 2cos[(a+b)/2]cos[(a-b)/2]
cosa-cosb= -2sen[(a+b)/2]sen[(a-b)/2]

Sei que dá para deduzí-las das outras, mas é trabalhoso.
Para decorar sen(a+b) e sen(a-b) eu usei um "poeminha":

"Minha tera tem palmeiras onde canta o sabiá 
 seno a cosseno b seno b cosseno a 
o sinal que vai aqui é o mesmo que vai lá
Pro cosseno é diferente senão não vai acertar!" 

Sei que éidiota, mas às vezes ajuda.
[]s Ronaldo L. Alonso
		Yahoo! Acesso Grátis: Internet rápida e grátis. Instale o discador agora!

Re: [obm-l] Problema de trigonometria

2005-04-29 Por tôpico Eder Albuquerque
Vc tem de lembrar que 

sena+senb= 2sen[(a+b) / 2] cos[(a-b)/ 2]
sena-senb= 2sen[(a-b)/2]cos[(a+b)/2]
cosa+cosb= 2cos[(a+b)/2]cos[(a-b)/2]
cosa-cosb= -2sen[(a+b)/2]sen[(a-b)/2]

Tanto a "ida" quanto a "volta" serão utilizadas abaixo...


1/cos6 + 1/sen24 + 1/sen48 =

=( sen24 sen48 + cos6 sen48 + cos6 sen24 ) /( cos6 sen24 sen48 )
=(1/2) (-cos72+cos24+sen54+sen42+sen30+sen18) / ( cos6 sen24 sen48 )

veja que -cos72+sen18=0 e escreva cos24=sen66, daí

=(1/2) ( sen66 + sen54 + sen42 + sen30 ) / ( cos6 sen24 sen48)
=(1/2)( 2sen60cos6 + 2sen36cos6) / ( cos6 sen24 sen48)

Cancelando o cos6, vem

=(sen60+sen36) / (sen24 sen48)
=(2sen48cos12) / (2sen12cos12sen48)
=1 / sen12
Felipe Takiyama [EMAIL PROTECTED] wrote:
Ajudem-me com esta:Prove que 1/(cos6°)+1/(sen24°)+1/(sen48°)=1/(sen12°).___Promoção Mergulhou, ganhou! Ganhe prêmios navegando pelo discador Click 21 de 25/04 a 30/06.Cadastre-se agora www.click21.com.br/mergulhouganhou=Instruções para entrar na lista, sair da lista e usar a lista emhttp://www.mat.puc-rio.br/~nicolau/olimp/obm-l.html=
		Yahoo! Acesso Grátis: Internet rápida e grátis. Instale o discador agora!

RE: [obm-l] Livros para ITA/IME

2005-04-16 Por tôpico Eder Albuquerque
Para os interessados neste tópico:

O que eu acredito ser mais interessante na preparação pro ITA é:

1)Resolver provas anteriores (muito importante)
2)Fazer simulados e prestar alguns vestibulares ao longo do ano para se acostumar a fazer provas
3)Dicas de professores experientes sobre assuntos corriqueiros neste vestibular
4)Resolver questões de outras instituições, mas só as mais "interessantes"
5)Atentar para o que cai mais na prova (em mat, geometria analítica sempre é bem explorada), detalhes como questões fáceis que costumam aparecer na prova de matemática, sempre da 15-ésima ao 20-ésimo teste etc.veja esses detalhes,podem dar pontos preciosos na prova...

Livros ajudam?

Ajudam, mas acredito que não dá pra confiar que estudar determinada coleção garante tal matéria. No meu tempo de cursinho, fiquei na dúvida, mas vi que, realmente, n tem livro que dê conta do recado sozinho(alguns passam longe). As aulas no cursinho foram determinantes...

Conheço algumas pessoas que n fizeram nada disso e passaram, mas elas, definitivamente, não são a regra...

Se ainda sim, quiserem uma indicação:

Química: Brady(de nível superior, mas acessível), Feltre
Mat:  Matemática Elementar
Fis: Robortella(não mais publicado, só em sebos), Adir Moisés


Eder ( 2 ano do ITA) 
saulo bastos [EMAIL PROTECTED] wrote:
quimicaÇcarmo gallo net, geraldo camargo, ricardo feltre, brett, etcfisica, fundamentos da fisica, alicerces da fisica,matematica, todos, um abra~co, saulo.From: marcio aparecido <[EMAIL PROTECTED]>Reply-To: obm-l@mat.puc-rio.brTo: obm-l@mat.puc-rio.brSubject: [obm-l] Livros para ITA/IMEDate: Sat, 16 Apr 2005 00:20:52 -0300aeee galera tô estudando para as provas do ITA/IME, e venho aqui pedirpara que me indiquem alguns livros de qiímica física e matemática.Livros tanto para pegar uma base bem sólida como tmb para um beloaprofundamento=Instruções para entrar na lista, sair da lista e usar a lista
 emhttp://www.mat.puc-rio.br/~nicolau/olimp/obm-l.html=_MSN Messenger: converse online com seus amigos . http://messenger.msn.com.br=Instruções para entrar na lista, sair da lista e usar a lista emhttp://www.mat.puc-rio.br/~nicolau/olimp/obm-l.html=
		Yahoo! Acesso Grátis: Internet rápida e grátis. Instale o discador agora!

[obm-l] relação de equivalência

2005-03-07 Por tôpico Eder Albuquerque
Olá a todos.

Eu gostaria de ajuda no seguinte problema:

Provar que a relação entre duas ou mais curvas que têm mesma orientação é uma relação de equivalência.

Obrigado.

Eder
		Yahoo! Acesso Grátis - Internet rápida e grátis. Instale o discador do Yahoo! agora.

[obm-l] curvas

2005-02-15 Por tôpico Eder Albuquerque
O que significa pendente de uma curva?

Vi esse termo no Demidovich (se tiver certa a escrita)...
		Yahoo! Acesso Grátis - Internet rápida e grátis. Instale o discador do Yahoo! agora.

Re: [obm-l] mais um de conjuntos

2005-02-15 Por tôpico Eder Albuquerque

[P' U (P inter Q)] = [P' U P] inter [P' U Q] = (conjunto universo)inter [P'U Q] = [P' U Q] 

Não seria alternativa "d"? Errei algo?




		Yahoo! Acesso Grátis - Internet rápida e grátis. Instale o discador do Yahoo! agora.

Re:[obm-l] Ajuda!! E conselhos!!!

2003-10-29 Por tôpico Eder

nb=2^(n)*1000,onde

nb=número de bactérias
n=tempo decorrido em horas a partir do momento em que temos 
1000 bactérias

Substituindo...

10^9=2^(n)*10^3

2^n=10^6

Aplicando log nos dois lados:

log(2^n)=6

nlog2=6

n=6/log2 = n=20h (aproximadamente...fiz na minha calculadora 
científica...)



 O número de bactérias numa certa cultura duplica a cada hora
. Se, num determinado instante, a cultura tem 1000 bactérias, 
então, o tempo aproximado, em horas, em que a cultura terá 1 b
ilhão de bactérias, é de?
  
 Eu fiz essa conta, manualmente, duplicando por hora.
 
 Mas eu sei que é possível utilizar se de logaritmos para res
olução, mas como faço isso?
  
 
-
  
 Eu queria que vcs me dessem sugestões, porque com exercícios
 que envolve Matemática no cotidiano, eu sinto tanta dificulda
de para interpreta-lo?
 
 Eu sinto que tenho um certa dificuldade nesses casos. Existe
 algo que possa me ajudar?
 
 
 
 -
 Yahoo! Mail - o melhor webmail do Brasil. Saiba mais!

 
---
Acabe com aquelas janelinhas que pulam na sua tela.
AntiPop-up UOL - É grátis! 
http://antipopup.uol.com.br

=
Instruções para entrar na lista, sair da lista e usar a lista em
http://www.mat.puc-rio.br/~nicolau/olimp/obm-l.html
=


Re:[obm-l] Livros!!

2003-10-24 Por tôpico Eder
Olá,

Eu entrei no ITA neste ano...É os seguinte:


1)A coleção do Iezzi é de fato uma ótima coleção,mas não é 
suficiente,acredito eu.

2)Não sei se você recebe no colégio,mas procure sempre 
os piores exercícios.Eu costumava receber muitos deste no 
cursinho,além de muitas,mas muitas mesmo,provas anteriores do 
ITA e do IME.RESOLVER PROVAS ANTERIORES É FUNDAMENTAL.

3)Aproveite essa lista.Há muita gente boa no assunto por 
aqui.Eu a acompanho desde 2001...Pergunte,acompanhe as 
discussões,veja como o pessoal ataca os problemas...

4)Vale a pena resolver a parte de exatas da FUVEST,por 
exemplo,principalmente as provas de segunda fase.Ah,procure 
questões  de outras instituições militares,tais como AFA e 
Escola Naval.Eu vivia baixando prova da AFA e tentando 
resolver no menor tempo possível pra testar minha velocidade.

Mais ou menos isso...
  
 Eu queria saber, o livros vocês aconselham de Matemática.
 E se der, tive intercalar em Iniciantes, Intermediário e Ava
nçado.
  
 Estou estudando para o Vestibular do ITA!!!
  
 Estudo atraves da Coleção do IEZZI, e uso os livros de Manoe
l Paiva para complementar.
 
 Mas é bom sempre ter outras fontes de consultas.
  
 Outra coisinha também, eu até que consigo resolver com facil
idades os exercícios da coleção do IEZZI, mas aparentemente es
tá um pouco longe da complexidade dos exercicios do ITA? Isso 
é normal? ou existem livros que tenha um envasamento mais cien
tífico.
 
 Desde de já agradeço,
  
 E tenha um Ótimo fim de semana!!!
 
 CARLOS
 
 
 
 -
 Yahoo! Mail - o melhor webmail do Brasil. Saiba mais!

 
---
Acabe com aquelas janelinhas que pulam na sua tela.
AntiPop-up UOL - É grátis! 
http://antipopup.uol.com.br

=
Instruções para entrar na lista, sair da lista e usar a lista em
http://www.mat.puc-rio.br/~nicolau/olimp/obm-l.html
=


Re:[obm-l] Livros!!

2003-10-24 Por tôpico Eder
Desculpa,acabei não dando sugestões de livros...Bem,eu tenho 
a coleção matemática do ensino médio da SBM (sociedade 
brasileira de matemática,www,sbm.org.br),além de outros 
livros tb da SBM.Uns livros da famosa editora Mir (Solving 
Problems in Algebra and Trigonometry,Análise de Funções 
Elementares,Geometry,Problems in Elementary 
Mathematics...).Lia e ainda leio as Eurekas.Tudo isso além do 
material do cursinho e da coleção do Iezzi.Não dei conta de 
tudo isso,fiz o que pude,mas felizmente fui bem no vestibular 
do ITA e passei.É meio difícil recomendar,tipo,muitos me 
diziam que era perda de tempo,mas funcionou pra mim,achei de 
fundamental importância esses estudos extras.Veja bem o que 
é o melhor pra vc.Eu já vi gente nunca pegar esses extras e 
passar no ITA,mas eu queria me garantir...

Falou,

Eder

 Olá,
 
 Eu entrei no ITA neste ano...É os seguinte:
 
 
 1)A coleção do Iezzi é de fato uma ótima coleção,mas não é 
 suficiente,acredito eu.
 
 2)Não sei se você recebe no colégio,mas procure sempre 
 os piores exercícios.Eu costumava receber muitos deste no 
 cursinho,além de muitas,mas muitas mesmo,provas anteriores d
o 
 ITA e do IME.RESOLVER PROVAS ANTERIORES É FUNDAMENTAL.
 
 3)Aproveite essa lista.Há muita gente boa no assunto por 
 aqui.Eu a acompanho desde 2001...Pergunte,acompanhe as 
 discussões,veja como o pessoal ataca os problemas...
 
 4)Vale a pena resolver a parte de exatas da FUVEST,por 
 exemplo,principalmente as provas de segunda fase.Ah,procure 
 questões  de outras instituições militares,tais como AFA e 
 Escola Naval.Eu vivia baixando prova da AFA e tentando 
 resolver no menor tempo possível pra testar minha velocidade
.
 
 Mais ou menos isso...
   
  Eu queria saber, o livros vocês aconselham de Matemática.
  E se der, tive intercalar em Iniciantes, Intermediário e A
va
 nçado.
   
  Estou estudando para o Vestibular do ITA!!!
   
  Estudo atraves da Coleção do IEZZI, e uso os livros de Man
oe
 l Paiva para complementar.
  
  Mas é bom sempre ter outras fontes de consultas.
   
  Outra coisinha também, eu até que consigo resolver com fac
il
 idades os exercícios da coleção do IEZZI, mas aparentemente 
es
 tá um pouco longe da complexidade dos exercicios do ITA? Iss
o 
 é normal? ou existem livros que tenha um envasamento mais ci
en
 tífico.
  
  Desde de já agradeço,
   
  E tenha um Ótimo fim de semana!!!
  
  CARLOS
  
  
  
  -
  Yahoo! Mail - o melhor webmail do Brasil. Saiba mais!
 
  
 ---
 Acabe com aquelas janelinhas que pulam na sua tela.
 AntiPop-up UOL - É grátis! 
 http://antipopup.uol.com.br
 
 
=
 Instruções para entrar na lista, sair da lista e usar a list
a em
 http://www.mat.puc-rio.br/~nicolau/olimp/obm-l.html
 
=
 

 
---
Acabe com aquelas janelinhas que pulam na sua tela.
AntiPop-up UOL - É grátis! 
http://antipopup.uol.com.br

=
Instruções para entrar na lista, sair da lista e usar a lista em
http://www.mat.puc-rio.br/~nicolau/olimp/obm-l.html
=


[obm-l] Re:[obm-l] Re: [obm-l] ciclo trigonométrico

2003-10-24 Por tôpico Eder

Note que 3cos(x) + 2sen(x)= 

sqrt(13)[ (3/sqrt(13)cosx+ 2/sqrt(13)senx] 

Não vou ser formal,mas veja que existe um ângulo y tal que 

seny=3/sqrt(13) e cosy=2(sqrt(13)  (relação 
fundamental...),daí ficamos com

sqrt(13)(senycosx+senxcosy) ou sqrt(13)sen(x+y)

Como o valor máximo de sen(x+y) é 1,o valor máximo de
sqrt(13)sen(x+y) é sqrt(13).

obs: sqrt(k) - raiz quadrada de k




 Não sei se entendi o problema direito, pois a solução que en
contrei foi
 muito simples:
 
 Como o maior valor do seno e qualquer ângulo é 1, f
(x) = 3.1+2.1 = 2.
 
 Se alguém tiver outra solução, me explique por favor...
 
 
 - Original Message - 
 From: Tiago Carvalho de Matos Marques [EMAIL PROTECTED]
.com.br
 To: [EMAIL PROTECTED]
 Sent: Friday, October 24, 2003 2:13 PM
 Subject: [obm-l] ciclo trigonométrico
 
 
  Qual o valor maximo da funcao f(x) = 3cos(x) + 2sen(x)?
  ==
===
  Instruções para entrar na lista, sair da lista e usar a li
sta em
  http://www.mat.puc-rio.br/~nicolau/olimp/obm-l.html
  ==
===
 
 
 
 
=
 Instruções para entrar na lista, sair da lista e usar a list
a em
 http://www.mat.puc-rio.br/~nicolau/olimp/obm-l.html
 
=
 

 
---
Acabe com aquelas janelinhas que pulam na sua tela.
AntiPop-up UOL - É grátis! 
http://antipopup.uol.com.br

=
Instruções para entrar na lista, sair da lista e usar a lista em
http://www.mat.puc-rio.br/~nicolau/olimp/obm-l.html
=


Re:[obm-l] Trigonometria III (Mr. Crowley)

2003-10-03 Por tôpico Eder
Ih,desculpa,é que eu já vi uma resolução para essa questão 
(acho que no matemática elementar...) e está como a sua,as 
diferenças são verificadas para tentar se notar alguma 
regularidade,aí achei que se estivesse fazendo uma 
suposição...


 Eder, voce nao pode sair supondo que tan
(B+C), etc estao em P.A pois e
 justamente o que voce tem que provar. 
 
  
 
  
 
 I) Sabendo que sen(2A), sen(2B) e sen(2C) estão em 
 
 P.A., nessa ordem, demonstrar que tan(B+C), tan(C+A) e 
 
 tan(A+B) também estão em P.A. nessa ordem. 
 
  
 
  
 
 Resolucao: 
 
  
 
 Seja (sin(2A),sin(2B),sin
(2C)) uma P.A de razao r, r0,  portanto, podemos
 escrever
 
  
 
 Sin(2B) - sin(2A) = r = 2cos(A+B).sin(B-A)  (1)
 
 Sin(2C) - sin(2B) = r = 2cos(C+B).sin(C-B)  (2)
 
 Sin(2C) - sin(2A) = 2r = 2cos(C+A).sin(C-A)(3)
 
  
 
 Vamos calcular as diferentas tan(A+B)-tan(C+A), tan(C+A)-tan
(B+C) e
 tan(A+B)-tan(B+C) e ver o que elas representam:
 
  
 
 *) tan(C+A)-tan(B+C) = (sin(C+A)/cos(C+A)) – (sin(B+C)/cos
(B+C)) .. Isolando
 cos(C+A) em (3) e cos(B+C) em (2) obtemos,
 
  
 
  = sin(C+A).sin(C-A)/r –
 (2.sin(B+C).sin(B-C))/r  , use o fato de
 cos(p)-cos(q)=-2.sin((p+q)/2).sin((p-
q)/2), logo, simplificando chegamos ao
 resultado,
 
  
 
  = (cos(2A)+cos(2C)-2cos
(2B))/2r  = K. 
 
  
 
 *) tan(A+B)-tan(C+A) = (sin(A+B)/cos(A+B))-(sin(C+A)/cos
(C+A)). Isolando as
 expressoes de cos(A+B) e cos
(A+C) em 1 e 3, respectivamente, obtemos,
 
  
 
 = (2.sin(A+B).sin(B-A))/r –
 (sin(C+A).sin(C-A))/r. Usando a formula de cos(p)-cos
(q) do item (*) temos
 
  
 
 = (cos(2A)-cos(2B))/r – (cos
(2A)-cos(2C))/2r
 
 
  
 
 = (cos(2A)+cos(2C)-2cos
(2B))/2r  = K.
 
  
 
 Analogamente, calcule agora tan(A+B)-tan
(B+C) e voce vai ver que encontrara
 
  
 
 tan(A+B)-tan(B+C) = 2K.
 
  
 
  
 
 Logo, tan(B+C),tan(C+A) e tan(A+B) estao em PA de razao
 K=[cos(2A)+cos(2C)-2cos(2B))]/2r , com r0. 
 
 
 
  
 
  
 
  
 
 Leandro L. Recova
 
  
 
 

 
---
Acabe com aquelas janelinhas que pulam na sua tela.
AntiPop-up UOL - É grátis! 
http://antipopup.uol.com.br

=
Instruções para entrar na lista, sair da lista e usar a lista em
http://www.mat.puc-rio.br/~nicolau/olimp/obm-l.html
=


Re:[obm-l] Trigonometria III (Mr. Crowley)

2003-10-02 Por tôpico Eder
Fala Mr. Crowley...

Sem querer ser grosso,apenas por curiosidade...Você pelo 
menos tentar resolver essas questões que você manda pra lista?
Cara,se você não tentar fazer sozinho,não vai aprender 
nunca,não adianta ficar só lendo resoluções.

Tô meio que com preguiça de escrever,então só vou te dizer 
pra lembrar que se sen(2A), sen(2B) e sen(2C) estão em 
PA,então sen(2b)=[sen(2A)+sen(2C]/2.De posse desse 
fato,suponha que tan(B+C), tan(C+A) e  tan(A+B) também estão 
em P.A.,nessa ordem, ou seja, o termo intermediário é média 
aritmética dos termos extremos,desenvolva a expressão e 
preste atenção no que vc vai chegar!

Para o outro,eu peguei uma resolução de alguém,por ter achado 
muito interessante.Juro que tentei pra caramba e não 
saiu.Olha só:


Como isso é um triangulo, entao A+B180 graus.

multiplicando a igualdade por  8cos(A/2).cos(B/2) (que
é diferente de zero
pois A,B  180 graus) temos :

sen(A/2) . [cos(B/2)]^3 = sen(B/2) . [cos(A/2)]^3
=
2.sen(A/2) .4. [cos(B/2)]^4. cos(A/2) = 2.sen(B/2) .
4.[cos(A/2)]^4
..cos(B/2)
=

lembrando que cos(2x)=cos²(x)-sen²(x)=2cos²(x)-1 =
cos(2x)=2cos²(x)-1 =
2cos²(x)=cos(2x)+1, fazendo x=B/2 temos: 2cos²(B/2)=cos(B)+1
substituindo para A e B  temos..

2.sen(A/2).cos(A/2).(cos(B)+1)²=2.sen(B/2).cos(B/2).(cos(A)+1)
²
=

como sen(2x)=2sen(x)cos(x) , fazendo x=A/2 temos : sen(A)=2sen
(A/2).cos(A/2)

substituindo temos:

sen(A).(cos(B)+1)²=sen(B).(cos(A)+1)²
=
sen(A).cos²(B)+2.sen(A).cos(B)+sen(A) =
sen(B).cos²(A)+2.sen(B).cos(A)+sen(B)
=
sen(A).cos²(B) - sen(B).cos²(A)+2(sen(A).cos(B)-sen(B).cos(A))
+sen(A)-sen(B)
= 0
=

como sen(A-B)=sen(A)cos(B)-sen(B).cos(A) entao:

sen(A).cos²(B) - sen(B).cos²(A)+2sen(A-B)+sen(A)-sen(B)
= 0
=

como cos²(x)=1-sen²(x) entao

2sen(A-B)+sen(A).(1-sen²(A))-sen(B).(1-sen²(B))+sen(A)-sen(B)
=0
=
2sen(A-B)+2sen(A)-2sen(B)-(sen³(A)-sen³(B))=0
= (*) explicação deste passo no final.
2sen(A-B)+(sen(A)-sen(B))(-3sen(A).sen(B)+2-(sen(A)-sen(B))²)
= 0
=
2sen(A-B)+(sen(A)-sen(B))(-3sen(A)sen(B)+2-(sen²(A)-2sen(A)sen
(B)+sen²(B)))
= 0
=
2sen(A-B)+(sen(A)-sen(B))(-3sen(A)sen(B)+2+2sen(A)sen(B)-1)
= 0
=
2sen(A-B)+(sen(A)-sen(B))(-sen(A)sen(B)+1) = 0

suponha 0BA180

entao..

2sen(A-B)0

(sen(A)-sen(B))0
é fácil ver que isso é verdade para A=90
se A=90+e, e0 e sen(B)=sen(A) então B=90-d,
0d=e, daí A+B=180+e-d =180,
o que é absurdo..

como sen(A)sen(B)=1 entao -sen(A)sen(B)+1=0

logo, a soma

2sen(A-B)+(sen(A)-sen(B))(-sen(A)sen(B)+1)

nunca pode ser zero, o que é absurdo .. pois com implicações
de = a partir
da hipótese chegamos que esta soma deve ser zero..
entao a hipótese de que AB é falsa.. pela simetria
do problema.. BA também
é falsa... então só pode ser A=B.

explicação do passo (*)

vou mostrar que 2x-2y - (x³-y³) = (x-y)(-3xy+2-(x-y)²)

sabemos que (x-y)³=x³+3xy²-3x²y-y³ = x³-y³-3xy(x-y)
= (x³-y³) =
(x-y)³+3xy(x-y) = (x-y)((x-y)²+3xy)

logo, 2(x-y) - (x³-y³) = (x-y)(2-(x-y)²-3xy) o que demonstra
a igualdade.



Falow's

Eder









 Olá Pessoal,
 
 Valew galera pelas ajudas! (Cláudio, Leandro, João, 
 Bruno e Ralph)
 
 
 Espero que possam me ajudar nestes dois também (que me 
 parece ser mais dificeis):
 
 I) Sabendo que 
 P.A., nessa ordem, demonstrar que tan(B+C), tan(C+A) e 
 tan(A+B) também estão em P.A. nessa ordem. 
 
 
 II) Demonstrar que é isósceles o triângulo ABC cujos 
 ângulos A e B verificam a equação 
 
 sen(A/2) . [cos(B/2)]^3 = sen(B/2) . [cos(A/2)]^3 
 
 
 
 Gostaria de aproveitar o espaço para perguntar se 
 alguém conhece algum site que tenha as resoluções das 
 provas do IME.
 
 É isso aí...
 
 Grato
 
 Mr. Crowley
 
(`-''-/).___..--''`-._   
 `6_ 6  )   `-.  ().`-.__.`)
 (_Y_.)'  ._   )  `._ `.``-..-' 
   _..`--'_..-_/  /--'_.' ,'   
  (il),-''  (li),'  ((!.-'
  
 
__
 Acabe com aquelas janelinhas que pulam na sua tela.
 AntiPop-up UOL - É grátis!
 http://antipopup.uol.com.br/
 
 
 
=
 Instruções para entrar na lista, sair da lista e usar a list
a em
 http://www.mat.puc-rio.br/~nicolau/olimp/obm-l.html
 
=
 

 
---
Acabe com aquelas janelinhas que pulam na sua tela.
AntiPop-up UOL - É grátis! 
http://antipopup.uol.com.br

=
Instruções para entrar na lista, sair da lista e usar a lista em
http://www.mat.puc-rio.br/~nicolau/olimp/obm-l.html
=


Re: [obm-l] Conjuntos - Justificativa

2003-07-14 Por tôpico Eder



Olá,

Gostaria de me "intrometer" na discussão só para 
questionar uma coisa...Bom,a definição de conjunto não diz que,por 
exemplo,
{a,b,c}={a,a,b,c},sendo a repetição 
desnecessária?

Não lembro direito,mas acredito que sim.Nesse 
caso,só escreveríamos um dos máximos que vc citou e pronto,o conjunto 
admitiria um máximo.Corrijam-me,caso tenha me enganado.

  - Original Message - 
  From: 
  Leandro 
  Fernandes 
  To: [EMAIL PROTECTED] 
  Sent: Monday, July 14, 2003 3:23 AM
  Subject: Re: [obm-l] Conjuntos - 
  Justificativa
  
  Então você está dizendo que essa afirmativa é 
  falsa?
  
  Se um conjunto X possuir ao menos dois elementos 
  máximos e iguais, este conjunto não tem máximo. É isso? Como poderia 
  justificar isso?
  
  
- Original Message - 
From: 
Eduardo 
Casagrande Stabel 
To: [EMAIL PROTECTED] 
Sent: Sunday, July 13, 2003 10:12 
PM
Subject: Re: [obm-l] Conjuntos - 
Justificativa

José.

Um conjunto X tem um máximo se ele possui um 
elemento x que é maior ou igual a todos os outros elementos de X. 


Duda.

  - Original Message - 
  From: 
  Jose 
  Francisco Guimaraes Costa 
  To: [EMAIL PROTECTED] 
  Sent: Sunday, July 13, 2003 10:03 
  PM
  Subject: [obm-l] Conjuntos - 
  Justificativa
  
  Comentário de um não-matemático que às vezes 
  confunde definições com postulados com teoremas, sobre a pergunta original 
  do Leandro.
  
  Ora,se o conjunto é "limitado 
  superiormente",nenhum de seus elementos pode ser maior que o 
  limite superior. Logo,ele certamente tem um máximo (que é menor ou 
  igual ao limite), e isto seria um corolário.
  
  Falei bobagem?
  
  JF
  - Original Message - 
  From: "Eduardo Casagrande Stabel" 
  [EMAIL PROTECTED]
  To: [EMAIL PROTECTED]
  Sent: Sunday, July 13, 2003 9:20 
  PM
  Subject: Re: [obm-l] Conjuntos - 
  Justificativa
   Caro Leandro.  Este é o chamado 
  axioma do sup. É equivalente a muitos outros, e não costuma-se 
  demonstrá-lo e sim usá-lo como axioma. Se você ainda quiser 
  demonstrá-lo, terá de estabalecer todos os axiomas dos reais, isto é, os 
  que você está usando (ou o livro). Do contrário, fica impossível 
  ajudá-lo.  Abração! Duda.  From: 
  "Leandro Fernandes" [EMAIL PROTECTED]  Pessoal, não consigo dar uma justificativa 
  plausível para esta afirmação:   "Todo conjunto 
  não vazio de números racionais limitado superiormente tem  
  máximo"   Alguém tem alguma sugestão? 

Leandro


Re: [obm-l] duvida de gabarito

2003-07-14 Por tôpico Eder



Tudo bom?

Eu me distra e acabei usando um "a" (que no era 
muito conveniente)para representar o nmero tal que 
sena=b/(a+b)^(1/2) e cosa=a/(a+b)^(1/2) .
Mandei outra mensagem trocando o "a" pelo "k",vc deve ter visto...Ento 
vamos trabalhar com o "k":

senk=b/(a+b)^(1/2) e cosk=a/(a+b)^(1/2) .

"TENHO EM 
MENTE QUE AS DUAS PARCELAS ACIMA FORAM CONSTRUIDAS PELA PRIMEIRA RELACAO 
FUNDAMENTAL SEN(A)^2 + COS(B)^2 = 1, "

Isso mesmo...

, MAS 
PARA ISSO DEVEMOS TER s 

ena=b/(a+b)^(1/2) e 
  cosa=a/(a+b)^(1/2), QUE EH O QUE VC DEFINE ABAIXO. MINHA DUVIDA ESTA 
  AQUI. COMO ESTABELECEU ESTAS DUAS EQUACOES 
  ?
Bom,primeiro notemos que a e b no podem ser 
simultaneamente nulos nas condies do problema,esqueci-me dessa observao nas 
outras mensagens...Veja tambm que -1  = a/(a+b) = 1, bem como 
-1  = b/(b+a)  = 1 e eles verificam a relao fundamental.Ainda,temos 
tgk=b/a (das definies que fiz),supondo "a" diferente de0.Seja f: 
[-pi/2,pi/2] -- R, f(k)=tgk. fcil ver que f  bijetora nesse 
domnio,logo,sendo b/a real,existe k em [-pi/2,pi/2] tal que 
f(k)=tgk=b/a.Portanto,existe tal k...

DE ONDE VEIO ESTE SEN(X+A) ? 
OBS: SEI QUE SEN(X+A) = SENX*COSA + SENA*COSX, MAS QUAL A RELACAO COM O 
PROBLEMA ? 

f(x)=(a+b)^(1/2) * [ a/(a+b)^(1/2) 
*senx + b/(a+b)^(1/2) cosx] =
= f(x)=(a+b)^(1/2) * [ 
cosk*senx +senk* cosx]

E vc v de onde aparece 
sen(x+k)...O interessante  que -1 = sen(x+k)  = 1.O problema no quer 
que f esteja em [-1,1]?Ento,se (a+b)^(1/2)=1,conseguiremos isso.Os valores da 
alternativa "a" so satisfatrios.



  - Original Message - 
  From: 
  [EMAIL PROTECTED] 
  
  To: [EMAIL PROTECTED] 
  Sent: Sunday, July 13, 2003 9:11 PM
  Subject: Re: [obm-l] duvida de 
  gabarito
  Tive algumas duvidas, se o autor ou qualquer membro puder 
  me esclarecer agradeceria. Estao no corpo da mensagem. 
  Em uma mensagem de 13/7/2003 20:42:55 Hora padro 
  leste da Am. Sul, [EMAIL PROTECTED] escreveu: 
  
  Faa o seguinte: f(x) = asen(x) + bcos(x) 
= f(x)=(a+b)^(1/2) * [ a/(a+b)^(1/2) *senx + 
b/(a+b)^(1/2) cosx] (ATE AQUI TUDO BEM.) 
  N
  ote que [a/(a+b)^(1/2) ]+[b/(a+b)^(1/2)] = 1,ento existe 
"a" tal que TENHO EM MENTE QUE AS DUAS 
  PARCELAS ACIMA FORAM CONSTRUIDAS PELA PRIMEIRA RELACAO FUNDAMENTAL 
  SEN(A)^2 + COS(B)^2 = 1, MAS PARA ISSO DEVEMOS TER s
  ena=b/(a+b)^(1/2) e cosa=a/(a+b)^(1/2), QUE EH O 
QUE VC DEFINE ABAIXO. MINHA DUVIDA ESTA AQUI. COMO ESTABELECEU 
ESTAS DUAS EQUACOES ?s
  ena=b/(a+b)^(1/2) e cosa=a/(a+b)^(1/2) 
E,portanto,f(x)=(a+b)^(1/2) * sen(x+a).J temos 
que -1  = sen(x+a)  =1,ento DE ONDE VEIO ESTE SEN(X+A) ? OBS: 
  SEI QUE SEN(X+A) = SENX*COSA + SENA*COSX, MAS QUAL A RELACAO COM O PROBLEMA 
  ? se tivermos (a+b)^(1/2) = 1,we are 
  done. 
  A nica alternativa satisfazendo isso  a alternativa 
"a". 
MESMO COM DUVIDAS, GOSTEI 
  MUITO DA RESOLUCAO. 
  - Original Message - 
From: [EMAIL PROTECTED] To: [EMAIL PROTECTED] 
  Sent: Sunday, July 13, 2003 7:30 PM Subject: [obm-l] 
  duvida de gabarito Ola pessoal, Vejam a questao 
  abaixo: Para que o conjunto imagem da funcao f(x) = asen(x) + 
  bcos(x) esteja contido no intervalo[-1; 1] eh suficiente que a e b sejam, 
  respectivamente, iguais a: a)(raiz)3/2 e 1/2 b)(raiz)3/3 
  e(raiz)3 c)(raiz)3 e(raiz)3/3 d)1 e 1 e)(raiz)2/2 e(raiz)2 
  gabarito: a Duvida: Nao entendi por que o gabarito diz que 
  a alternativa a eh acorreta. Fiz diferente: Considerei a 
  expressao: asen(x) +bcos(x) em dois casos asen(x) + bcos(x)= -1 e 
  asen(x) + bcos(x)= -1 Ateh agora naofiz nada de novo, apenas 
  equacionei o que o enunciado diz: ... -1 = f(x) 
  = 1 (intervalos inclusos oo e +oo) Para que as 
  duasequacoes facam sentido devemos fazer x= pi/2, pi, 3pi/2 e/ou 
  2pi, pois estes arcos possuem senos ecossenos extremos. Inspecionando 
  verificamos que a= 1 ou -1 e b= -1 ou -1 tbem.Para satisfazer as 
  alternativas devemos considerar a=1, b=1. Alternativa d. O que fiz de 
  errado ? 


Re: [obm-l] Re: [obm-l] Combinatoria

2003-07-14 Por tôpico Eder



Olá,

Já vi esse princípio também como "princípio das 
gavetas".A idéia é simples,porém pooderosa na resolução de alguns problemas.Se 
vc tem n+1 objetos para distribuir em n gavetas,então vc pode afirmar com 
certeza que pelo menos uma gaveta possui mais de 
umobjeto.É bem interesante...Quantas pessoas, no mínimo, vc 
tem de reunirpara ter certeza de que pelo menos duas nasceram num mesmo 
dia da semana (dom,seg,...,sáb)?Ora,se vc juntar 7, pode acontecer,num caso 
extremo,de cada uma ter nascido num dia diferente.Juntando 8,com certeza,pelo 
menos duas terão nascido num mesmo dia.Vc mesmo pode pensar em mais situações 
para a aplicação desse princípio.Bom deixar claro que ele não serve apenas para 
responder a essas "perguntinhas".Vc pode usar em situações como a do problema 
abaixo e já vi outras aplicações interessantes no Problem Solving 
Strategies...



  - Original Message - 
  From: 
  Rafael 
  Ando 
  To: [EMAIL PROTECTED] 
  Sent: Monday, July 14, 2003 3:40 AM
  Subject: Re: [obm-l] Re: [obm-l] 
  Combinatoria
  Alguem me explica como eh esse principio da casa dos 
  pombos?obrigadoFrom: [EMAIL PROTECTED]Reply-To: 
  [EMAIL PROTECTED]To: [EMAIL PROTECTED]Subject: 
  [obm-l] Re: [obm-l] CombinatoriaDate: Sat, 12 Jul 2003 12:22:50 
  -0300 Oi Marcio, Se eu não 
  me engano, esse problema tem no Problem Solving: Seja x_i= 
  número de partidas jogadas até o dia i, inclusive. Como o 
  enxadrista joga no minimo 1 partida por dia e no máximo 11x12=132 no 
  total, temos 1= a_1 a_2... a_77= 132. 
  Some 20 na desigualdade: 21= a_1 + 20... a_77 + 20 
  = 152. Então, os números a_1, a_2,..., a_77, a_1 + 
  20,...,a_77 + 20 estão entre1 e 152. Como temos 154 números, pelo 
  princípio da casa dos pombos existemdois deles iguais. Assim, existem 
  dois indices i e j, i!=j, tais 
  que 
  a_i= a_j + 20. Ora, isso é equivalente ao enxadrista ter 
  jogado exatamente vinte partidasentre os dias i+1 e 
  j. Ateh mais, Yuri-- 
  Mensagem original --  Nao estou 
  conseguindo fazer a seguinte questao, do livro de combinatoria 
  do Morgado: Um enxadrista joga partidas de xadrez durante onze 
  semanas consecutivas. Sabe-se que ele sempre joga ao menos uma 
  partida por dia, e jamais jogamais de 12 partidas em uma 
  semana. Mostre que existe um periodo de dias consecutivos 
  no qual ele joga exatamente 20 partidas.  
  Alguem tem alguma dica?   
  Abracos,  Marcio 
  []'s, YuriICQ: 
  64992515--Use 
  o melhor sistema de busca da InternetRadar UOL - http://www.radaruol.com.br=Instruções 
  para entrar na lista, sair da lista e usar a lista 
  emhttp://www.mat.puc-rio.br/~nicolau/olimp/obm-l.html=_MSN 
  Hotmail, o maior webmail do Brasil. http://www.hotmail.com=Instruções 
  para entrar na lista, sair da lista e usar a lista emhttp://www.mat.puc-rio.br/~nicolau/olimp/obm-l.html=


Re: [obm-l] ALGEBRA VETORIAL

2003-07-13 Por tôpico Eder



Talvez eu vá repetir algo,mas vamos 
lá:

Sejam o espaço vetorial S={u1,u2,u3,...un} e 
V={w1,w2,w3,...,wm} um conjunto tal que qualquer uj pertencente a S pode ser 
escrito como uma combinação linear dos elementos de V.Assim,V gera S,ou 
seja,S=[V].Quando acontecer de Vestar contido emS,ou seja, cada 
wifor igual a um certo uj,V continuar gerando S e V for um conjunto 
linearmente independente,então V é uma base de S.

A grosso modo,diria que V é uma base de S quando 
possui os vetores "fundamentais" para a "construção" de qualquer vetor do 
conjunto S que vc queira .É como se os vetores deV fossem as "cores 
fundamentais" a partir dos quais obtemos todas as outras "cores" (elementos de 
S).

Espero ter ajudado.

Eder

  - Original Message - 
  From: 
  Felipe Gastaldo 
  To: [EMAIL PROTECTED] 
  Sent: Sunday, July 13, 2003 12:07 
PM
  Subject: [obm-l] ALGEBRA VETORIAL
  Caros colegas da lista eu estou tendo um curso dealgebra 
  vetorial e o professor definiu BASE, mas eunaum consigo entender, já 
  li a definição do livroApostol e tb naum entendi gostaria que alguem 
  pudesseme dar uma definição clara e simples sobre BASE.muito obrigado 
  Felipe Gastaldo 
  ___Yahoo! 
  MailMais espaço, mais segurança e gratuito: caixa postal de 6MB, 
  antivírus, proteção contra spam.http://br.mail.yahoo.com/=Instruções 
  para entrar na lista, sair da lista e usar a lista emhttp://www.mat.puc-rio.br/~nicolau/olimp/obm-l.html=


Re: [obm-l] duvida de gabarito

2003-07-13 Por tôpico Eder



Faa o seguinte:


f(x) = asen(x) + bcos(x) = 
f(x)=(a+b)^(1/2) * [ a/(a+b)^(1/2) *senx + b/(a+b)^(1/2) 
cosx]

Note que [a/(a+b)^(1/2) ]+[b/(a+b)^(1/2)] = 
1,ento existe "a" tal que

sena=b/(a+b)^(1/2) e cosa=a/(a+b)^(1/2) 


E,portanto,f(x)=(a+b)^(1/2) * sen(x+a).J temos que -1  = sen(x+a)  
=1,ento se tivermos (a+b)^(1/2) = 1,we are 
done.
A nica alternativa satisfazendo isso  a 
alternativa "a".



- Original Message - 

  From: 
  [EMAIL PROTECTED] 
  
  To: [EMAIL PROTECTED] 
  Sent: Sunday, July 13, 2003 7:30 PM
  Subject: [obm-l] duvida de gabarito
  Ola pessoal, 
  Vejam a questao abaixo: Para que o conjunto imagem da 
  funcao f(x) = asen(x) + bcos(x) esteja contido no intervalo[-1; 1] eh 
  suficiente que a e b sejam, respectivamente, iguais a: a)(raiz)3/2 e 
  1/2 b)(raiz)3/3 e(raiz)3 c)(raiz)3 e(raiz)3/3 d)1 e 1 
  e)(raiz)2/2 e(raiz)2 gabarito: a Duvida: Nao entendi por 
  que o gabarito diz que a alternativa a eh acorreta. Fiz diferente: 
  Considerei a expressao: asen(x) +bcos(x) em dois casos asen(x) + 
  bcos(x)= -1 e asen(x) + bcos(x)= -1 Ateh agora naofiz nada de 
  novo, apenas equacionei o que o enunciado diz: ... -1 = f(x) = 1 
  (intervalos inclusos oo e +oo) Para que as duasequacoes facam 
  sentido devemos fazer x= pi/2, pi, 3pi/2 e/ou 2pi, pois estes arcos 
  possuem senos ecossenos extremos. Inspecionando verificamos que a= 1 ou -1 e 
  b= -1 ou -1 tbem.Para satisfazer as alternativas devemos considerar a=1, b=1. 
  Alternativa d. O que fiz de errado ? 



[obm-l] Re: [obm-l] análise de sinais (funções)

2003-01-16 Por tôpico Eder



Chamemos de C o complementar de B em relação a 
R.Calculemos B:

x^2 - 4x + 3  0 = (x-1)(x-3)  0 
= x 1 ou x3 ,ou seja, B=(-inf,1) U (3,+inf)

Por conseguinte,teremos C=[1,3].

Cálculo de A:

x^2 - 3x + 2 = 0 = (x-1)(x-2) 
 = 0 = 1 = x  =2,ou seja, 
A=[1,2]

Note que A está contido em C,logo A 
interseção C será igual a A,isto é, [1,2].



- Original Message - 

  From: 
  [EMAIL PROTECTED] 
  
  To: [EMAIL PROTECTED] 
  Sent: Wednesday, January 15, 2003 8:16 
  PM
  Subject: [obm-l] análise de sinais 
  (funções)
  Olá pessoal, 
  Como posso resolver está questão: (PUC-SP) Se A= {x 
  pertencendo à R, tal que x^2 - 3x + 2 = 0 } e B= {x pertencendo à R, tal 
  que x^2 - 4x + 3  0} então (A intersecção com B), onde B é o complementar 
  de B em relação a R, é igual a :  Resp: {x pertencendo à R, tal 
  que 1=x=2} Como chegar neste resultado? 



[obm-l] geometria

2003-01-16 Por tôpico Eder



Gostaria de ajuda no problema abaixo:


Considere duas circunferências tangentes 
internamente em um ponto A.Traça-se uma corda BCna maior circunferência de 
modo que essa corda tangencie a menor circunferência num ponto D.Prove que a 
semi-reta AD é bisssetriz do ângulo BAC.

Eu fiz o esquema aqui,mas não consegui 
provar...

Eder


[no subject]

2003-01-16 Por tôpico Eder



Mais uma dúvida:
R é um 
retângulo.Encontre o conjunto de todos os pontos que estão mais próximos do 
centro do retângulo que de qualquer vértice.
Agradeço 
comentários.

Eder



Re: [obm-l] Conjuntos finitos

2003-01-15 Por tôpico Eder



Dadoum elemento qualquer pertencente a X,temos n 
posssibilidades de correspondência com algum outroelemento de Y.Pode-se 
dizer o mesmo para demais elementos de X.Daí,o total de sequências 
depares ordenados que podem ser formadas será n*n*n...*n,m 
vezes,sendo que cada sequência de pares ordenados representa um função em 
particular.Acho que isso mostra que cardF(X,Y)=n^m
Corrijam-me se eu tiver cometido algum equívoco.

Eder

  - Original Message - 
  From: 
  Tertuliano 
  Carneiro de Souza Neto 
  To: [EMAIL PROTECTED] 
  Sent: Wednesday, January 15, 2003 5:05 
  PM
  Subject: [obm-l] Conjuntos finitos
  Olá pessoal, nao consegui resolver o problema abaixo.Alguem 
  pode tentar pra mim, por favor?Seja F(X;Y) o conjunto das 
  funcoes com dominio em X eimagem em Y. Se cardX=m e cardY=n, prove 
  que 
  cardF(X;Y)=n^m.Tertuliano Carneiro.De 
  Salvador. 
  ___Yahoo! 
  GeoCitiesTudo para criar o seu site: ferramentas fáceis de usar, espaço de 
  sobra e acessórios.http://br.geocities.yahoo.com/=Instruções 
  para entrar na lista, sair da lista e usar a lista emhttp://www.mat.puc-rio.br/~nicolau/olimp/obm-l.htmlO 
  administrador desta lista é [EMAIL PROTECTED]=


[obm-l] Re: [obm-l] Equação

2003-01-13 Por tôpico Eder



Para x diferente de 1,3 e 5,a equação é equivalente 
a

25*(x-1)*(x-2)*(x-3)*(x-4) = 
x*(x-1)*(x-2)*(x-3)*(x-4)

ou ainda


25*(x-2)*(x-4) = 
x*(x-2)*(x-4)

Facilmente,vê-se que x=2 e x=4 satisfazem a relação 
acima.Finalmente,para x diferente de 2 e de 4,resta

x=25.

Logo,são três valores satisfatórios: 2,4 e 
25.


  - Original Message - 
  From: 
  [EMAIL PROTECTED] 
  
  To: [EMAIL PROTECTED] 
  Sent: Sunday, January 12, 2003 4:56 
  PM
  Subject: [obm-l] Equação
  Olá pessoal, 
  Porque o número de raízes da equação abaixo é 3 ? 
  [25*(x-1)*(x-2)*(x-3)*(x-4)] / [(x-1)*(x-3)*(x-5)] = 
  [x*(x-1)*(x-2)*(x-3)*(x-4)] / [ (x-1)*(x-3)*(x-5)] Dúvida: Eu 
  "cortei" o (x-1)*(x-2)*(x-3)*(x-4)] / [(x-1)*(x-3)*(x-5)] nos dois membros e 
  obtive x=25, não está certo? Como chegar ao resultado de 3 raízes ? 
   


[obm-l] Re: [obm-l] Geometria analtica

2003-01-13 Por tôpico Eder



Vc tem de levar em consideração que o ponto M 
pertence à reta y-2x+5 = 0e à circunferência x²+y²=5 ao mesmo 
tempo.
Para y= 1,na reta,vem que1-2x+5=0 = 
x=3 e teríamos o ponto (3,1),diferente de M...

  - Original Message - 
  From: 
  [EMAIL PROTECTED] 
  
  To: [EMAIL PROTECTED] 
  Sent: Sunday, January 12, 2003 5:07 
  AM
  Subject: [obm-l] Geometria 
analítica
  Olá pessoal, 
  Vejam a questão: A reta y – 2x + 5 = 0 
  tangencia, no ponto M, a circunferência C de equação x^2 + 
  y^2 = 5. A reta y = – x + p intercepta C nos pontos 
  M e Q. Determine o valor de p ? Resolução: 
  y = 2x - 5 x^2 
  + y^2 = 5 , portanto x^2 + (2x – 5)^2 = 5x^2+ 4x^2 – 20x + 20 = 0 
  5 x^2 – 20x + 20 = 0  x= 2 y= – 1 Logo, M 
  (2,– 
  1)y = – x + p 
  – 1 = – 2 + p 
   p = 1  
  Dúvida: Substituindo o x=2 em x^2+ y^2 = 5 temos y=1 
  e y= -1, então porque só foi considerado o -1 em M (2,– 1)? Por que não considerar 
  M (2, 1) se a eq. do 2º da dois valores o -1 e também o 1? 

  


Re: [obm-l] Problema t

2003-01-10 Por tôpico Eder



cos(p/65).cos(2p/65).cos(4p/65).cos(8p/65).cos(16p/65).cos(32p/65) =x
sen(pi/65)cos(p/65).cos(2p/65).cos(4p/65).cos(8p/65).cos(16p/65).cos(32p/65) =xsen(pi/65)
(1/2)sen(2pi/65)cos(2p/65).cos(4p/65).cos(8p/65).cos(16p/65).cos(32p/65)=xsen(pi/65)
(1/4)sen(4pi/65)cos(4p/65).cos(8p/65).cos(16p/65).cos(32p/65)=xsen(pi/65)
(1/8)sen(8pi/65)cos(8p/65).cos(16p/65).cos(32p/65) =xsen(pi/65)
(1/16)sen(16pi/65).cos(16p/65).cos(32p/65) =xsen(pi/65)
(1/32)sen(32pi/65)cos(32p/65)=xsen(pi/65)
(1/64)sen(64pi/65)=xsen(pi/65)

Note agora que 64pi/65 e pi/65 são suplementares,ou 
seja,sen(64pi/65)=sen(pi/65).Daí:

x=1/64 

Se eu não tiver me atrapalhado em alguma etapa,é isso aí.

Eder

  - Original Message - 
  From: 
  Bruno 
  To: OBM-L 
  Cc: OBM-L 
  Sent: Friday, January 10, 2003 7:34 
  PM
  Subject: [obm-l] Problema "t"
  
  Olá pessoal,
  Eu estava tentando este problema e não 
  conseguiu.
  "cos(p/65).cos(2p/65).cos(4p/65).cos(8p/65).cos(16p/65).cos(32p/65) é igual a:
  a)1/2
  b)1/8
  c)1/32
  d)1/64
  e)1 "
  Até


Re: [obm-l] geometria plana

2003-01-09 Por tôpico Eder



Vou tentar descrever direitinho.Essa saiu até 
facilemente.

Vamos aplicar o Teorema da Bissetriz interna duas 
vezes.Lembrando:

Num triângulo ABC,seja AD a bissetriz do ângulo Â,D 
sobre BC.Então vale:

BD/AB = CD/AC

Beleza?

Então consideremos agoranossa 
situação.Chamemos AS de x e CS de 7-x.Pelo TBI:

x/6 = (7-x)/8 (*)

Condidere agora o triângulo 
CBS.Novamente:


8/BI = (7-X)/IS == BI/IS = 8(7-x) 
(**)

Só que de (*),por uma propriedade das 
proporções:

x/6 = (7-x)/8 = (x+7-x)/(6+8) = 7/14 = 
1/2

Ou seja, 8/(7-x)=1/2 (***).Comparando (***) e 
(**),temos o resultado desejado.





  - Original Message - 
  From: 
  [EMAIL PROTECTED] 
  
  To: [EMAIL PROTECTED] 
  Sent: Thursday, January 09, 2003 8:06 
  PM
  Subject: [obm-l] geometria plana
  Olá pessoal, Imaginem um triângulo de base BC=8, AB= 6, AC= 7. 
  Sendo BS bissetriz do ângulo B ( o ponto S pertence à AC) e CI bissetriz do 
  ângulo C (o ponto I é o ponto de intersecção das bissetrizes). Como eu posso 
  provar que a razão BI/IS vale 2 ? 


Re: [obm-l] geometria plana

2003-01-09 Por tôpico Eder



Droga...Cometi um erro na passagem "Ou seja, 8/(7-x)=1/2 (***).Comparando (***) e (**),temos o 
resultado desejado."

Como vc mesmo pode ver,é 8/(7-x)=2.Foi mal 
aí.

  - Original Message - 
  From: 
  Eder 
  To: [EMAIL PROTECTED] 
  Sent: Thursday, January 09, 2003 8:14 
  PM
  Subject: Re: [obm-l] geometria 
plana
  
  Vou tentar descrever direitinho.Essa saiu até 
  facilemente.
  
  Vamos aplicar o Teorema da Bissetriz interna duas 
  vezes.Lembrando:
  
  Num triângulo ABC,seja AD a bissetriz do ângulo 
  Â,D sobre BC.Então vale:
  
  BD/AB = CD/AC
  
  Beleza?
  
  Então consideremos agoranossa 
  situação.Chamemos AS de x e CS de 7-x.Pelo TBI:
  
  x/6 = (7-x)/8 (*)
  
  Condidere agora o triângulo 
  CBS.Novamente:
  
  
  8/BI = (7-X)/IS == BI/IS = 8(7-x) 
  (**)
  
  Só que de (*),por uma propriedade das 
  proporções:
  
  x/6 = (7-x)/8 = (x+7-x)/(6+8) = 7/14 = 
  1/2
  
  Ou seja, 8/(7-x)=1/2 (***).Comparando (***) e 
  (**),temos o resultado desejado.
  
  
  
  
  
- Original Message - 
From: 
[EMAIL PROTECTED] 
To: [EMAIL PROTECTED] 
Sent: Thursday, January 09, 2003 8:06 
PM
Subject: [obm-l] geometria plana
Olá pessoal, Imaginem um triângulo 
de base BC=8, AB= 6, AC= 7. Sendo BS bissetriz do ângulo B ( o ponto S 
pertence à AC) e CI bissetriz do ângulo C (o ponto I é o ponto de 
intersecção das bissetrizes). Como eu posso provar que a razão BI/IS vale 2 
? 


Re: [obm-l] Problemas de Geometria

2003-01-06 Por tôpico Eder
Title: Help



Obrigado pelas dicas e pelos problemas 
propostos,Cláudio.Eu nunca fiz aula de preparação para olimpíada e tenho ralado 
sozinho mesmo.Só fui me interessar mais pelo assunto depois que concluí o ensino 
médio e por causa de uns vestibulares difíceis que tentei com sucesso(ITA e 
IME).Só me resta agora participar do nível universitário.O problema é que as 
questões são bem difíceis,mais dirigidas a quem vinha fazendo olimpíadas 
continuamente...Mas acho que vou participar,não custa nada.Valeu!

  - Original Message - 
  From: 
  Cláudio (Prática) 
  To: [EMAIL PROTECTED] 
  Sent: Monday, January 06, 2003 5:00 
  PM
  Subject: [obm-l] Problemas de 
  Geometria
  
  
  Caro Eder:
  
  Pode acreditar que os seus dois problemas de 
  geometriaforem difíceis pra mim também. 
  
  Repare que nos dois problemas aparecem, de uma 
  forma ou de outra, ângulos inscritos em circunferências. Na maioria dos 
  problemas envolvendo ângulos vale a pena checar para ver se alguma 
  circunferência contém dois ou mais dos ângulos do problema ou, como no caso do 
  problema 1, se você pode transladar algum ângulo de forma que ele fique 
  inscrito na mesma circunferência que algum outro.
  
  Outros itens que aparecem com frequência e são a 
  chave para a solução do problema são quadriláteros inscritíveis, 
  paralelogramos, triângulos isósceles esemelhança de triângulos. Não 
  existe um método fixopara se atacar problemas de geometria 
  (especialmente a nível de olimpíada). No entanto, há uma grande probabilidade 
  que estes problemas envolvam os elementos acima.
  
  O pior caso é quando você precisa construir uma 
  reta ou segmento auxiliar a fim de fazer um dos itens acima aparecer. Aí, acho 
  que só a experiência ajuda...
  
  Como treino, tente os seguintes 
  problemas:
  
  1) O triângulo ABC é isosceles, com AB = AC. O 
  ângulo BAC mede 20 graus. Traçam-se os segmento BD e CE,(D em AC e entre 
  A e C; E em AB e entre A e B) formando, com a base BC,ângulos de 60 e 50 
  graus, respectivamente. Calcule o valor do ângulo BDE. Dica: construa um 
  segmento auxiliar que faça aparecer triângulos isósceles ou, com sorte, um 
  triângulo equilátero.
  
  2) Prove o Teorema de Ptolomeu: Num quadrilátero 
  inscritível ABCD, vale AB*CD + AD*BC = AC*BD. Dica: um segmento auxiliar bem 
  construído podeproduzir triângulos 
  semelhantesestratégicos.
  
  3) Dado um triângulo ABC, construa três 
  circunferências tendo, cada uma, um dos lados do triângulo como diâmetro. 
  Prove que os pontos de interseção de cada par de circunferências pertence a 
  pelo menos um dos lados do triângulo. 
  
  4) No problema anterior, prove que as três cordas 
  que unem os pontos de interseção de cada parcircunferências são 
  concorrentes. Que ponto é esse (em relação ao triângulo ABC)?
  
  5) Um triângulo equilátero ABC está inscrito numa 
  circunferência. Prove que, qualquer que seja o ponto P no arco BC (que não 
  contém o vértice A), teremos: PA = PB + PC. Dica: use um dos problemas 
  anteriores.
  
  Um abraço,
  Claudio 
Buffara.


Re: [obm-l] geometria

2003-01-05 Por tôpico Eder



Ótimo.Eu juro que tentei pra caramba,mas não saia 
nada.Valeu!

  - Original Message - 
  From: 
  larryp 
  To: [EMAIL PROTECTED] 
  Sent: Sunday, January 05, 2003 9:04 
  PM
  Subject: Re: [obm-l] geometria
  
  Problema 2:
  
  ABCD 
  é um quadrilátero cíclico. Areta tangentepor A encontra CB 
  em K,e a reta tangentepor B encontra DA em M,de maneira que BK=BC e 
  AM=AD. Mostre que o quadrilátero tem dois lados 
  paralelos.
  
  O resultado estará provado se conseguirmos 
  mostrar que os ângulos MAB e MDC são iguais.
  
  1)Tome pontos L em AK e N em MB tais que A 
  esteja entre K e L eque B esteja entre M e N e em seguida use as propriedades dos ângulos e arcos na 
  circunferência:
  
  Arco AB = KAB = MDB = KCA = MBA
  
  Arco ADC = ABC
  
  Arco BCD = BAD
  
  
  2) Com as igualdades de ângulos deduzidas acima, 
  conclua que certos triângulos são semelhantes:
  
  KCA = KABe CKA = AKB == 
  Triângulos KCA e KABsão semelhantes 
  ==
  AC / AB = KC / KA = KA / KB == 
  KA^2 = KB * KC
  
  MDB = MBA e DMB = BMA ==
  Triângulos MBD e MAB são semelhantes 
  ==
  BD / AB= MD / MB = MB / MA 
  == MB^2 = MA * MD
  
  
  3) Levando em conta que KC = 2 * KB e MD = 2 * 
  MA, teremos:
  
  KA^2 = 2 * KB^2 e MB^2 = 2 * 
  MA^2 ==
  KA / KB = MB / MA = 
  raiz(2)
  
  Ou seja, AC / AB = KA / KB = MB / MA = BD / AB = 
  raiz(2)
  
  Assim, AC = BD = AB * raiz(2)
  
  
  4) Cordas iguais subentendem arcos iguais. 
  
  AC = BD == Arco ADC = Arco BCD 
  ==
  Ângulo ABC = Ângulo BAD.
  
  
  5) ABCD é cíclico == 
  ABC + CDA = 180 graus ==
  BAD + CDA = 180 graus
  
  Mas, MAB + BAD = 180 graus (são suplementares) 
  ==
  MAB = CDA = MDC ==
  AB // CD e o resultado está provado.
  
  
- Original Message - 
From: 
Eder 
To: [EMAIL PROTECTED] 
Sent: Thursday, January 02, 2003 2:51 
PM
Subject: [obm-l] geometria

Doisproblemas que não estou conseguindo 
resolver:

1)ABCD 
é um quadrilátero.M é um ponto interno a esse quadrilátero de forma que ABMD 
é um paralelogramo.O ângulo CBM é igual ao ângulo CDM.Mostre que o ângulo 
ACD é igual ao ângulo BCM.

2)ABCD 
é um quadrilátero cíclico.Areta tangentepor A encontra CB 
em K,e a reta tangentepor B encontra DA em M,de maneira que BK=BC e 
AM=AD.Mostre que o quadrilátero tem dois lados 
paralelos.

Qualquer 
ajuda/resolução é bem vinda.


Eder


Re: [obm-l] Trigonometria

2003-01-03 Por tôpico Eder



a)sen(27pi)=sen(26pi+pi)=senpi=0

Em geral,sen(teta+2kpi)=sen(teta) com k inteiro.São 
todos arcos côngruos...

b)sen(-37pi3)=sen(-36pi/3 -pi3)=sen(-12pi 
-pi/3)=sen(-pi/3)=-sen(pi/3)= -sqrt3/2

c)sen(15pi/2)=sen(7pi 
+pi/2)=sen(pi+pi/2)=sen(3pi/2)= -1





  - Original Message - 
  From: 
  [EMAIL PROTECTED] 
  
  To: [EMAIL PROTECTED] 
  Sent: Friday, January 03, 2003 4:01 
  AM
  Subject: [obm-l] Trigonometria
  Por que tg(9pi/4) = 
  tg(pi/4) ? Como fazer estas conversões? Regra de três ou os conceitos de arcos 
  côngruos e/ou equações trigonométricas? Como calcular: a) sen 27pi 
  b) sen (-37pi/3) c)cos (15pi/2) Se vcs me explicarem como 
  fazer estes três eu ficarei muito agradecido, pois assim poderei fazer o 
  restante do meu caderno de estudos. Ps: O exercício que eu estou fazendo 
  tem + ou - 18, a maioria eu fiz utilizando regra de três, ou seja, 
  transformando os radianos em graus, dividindo por 360º e pegando o resto como 
  valor notável mas o problema é que este resto nem sempre dava um valor 
  notável. Será que meu erro está em transformar em graus? Devo fazer regra de 
  três de radianos para radianos, pois neste tipo de exercício como vocês 
  podem ver acima, pede para calcular o sen,cos e tg só de radianos e não de 
  graus. Em alguns casos eu até resolvi facilmente como sen (17pi/2) ou sen 
  (-13pi/2), pois encontrava valores notáveis. Mas, nos itens como em a, b,c não 
  encontrei esses valores notáveis. 


[obm-l] Re: [obm-l] Geometria analítica

2003-01-02 Por tôpico Eder



Seja P(a,b) o ponto médio do segmento 
AC.Calculam-se a e b facilmente:

a = (1+7)/2=4
b = (2+4)/2=3

Basta achar a equação da reta que passa por B(4,5) 
e por P(4,3).Como a reta será da formaax+by+c=0 e para x=4 temos dois 
valores correspondentes,tá na cara que só podemos ter a=1,b=0 e c=-4 ,ou 
seja,x=4.Na dúvida,tire a reta pelo método do determinante.

  - Original Message - 
  From: 
  [EMAIL PROTECTED] 
  
  To: [EMAIL PROTECTED] 
  Sent: Thursday, January 02, 2003 5:13 
  AM
  Subject: [obm-l] Geometria 
analítica
  
  Determine a equação da mediana relativa ao lado 
  AC de um triângulo cujos vértices são os pontos A(1,2) , B(4,5) e C(7,4). 
  Resposta 
  A equação da mediana é x=4. Como operar para chegar na 
  equação da mediana como nessa questão ? 
  


[obm-l] geometria

2003-01-02 Por tôpico Eder



Doisproblemas que não estou conseguindo 
resolver:

1)ABCD 
é um quadrilátero.M é um ponto interno a esse quadrilátero de forma que ABMD é 
um paralelogramo.O ângulo CBM é igual ao ângulo CDM.Mostre que o ângulo ACD é 
igual ao ângulo BCM.

2)ABCD 
é um quadrilátero cíclico.Areta tangentepor A encontra CB em 
K,e a reta tangentepor B encontra DA em M,de maneira que BK=BC e 
AM=AD.Mostre que o quadrilátero tem dois lados 
paralelos.

Qualquer 
ajuda/resolução é bem vinda.


Eder


[obm-l] Re: [obm-l] sistema de equaçoes

2003-01-02 Por tôpico Eder



Da primeira equação, vem que x=2 e y=3.O que é que 
resulta em 6?

  - Original Message - 
  From: 
  [EMAIL PROTECTED] 
  
  To: [EMAIL PROTECTED] 
  Sent: Thursday, January 02, 2003 7:16 
  PM
  Subject: [obm-l] sistema de 
equaçoes
  Uma com dúvida na 
  seguinte questão: 2^x *3^y=108 4^x*2^y=128 Ps: a resposta é 6 
  Eu não estou conseguindo resolver este sistema de equações pois sempre eu 
  "caio"em uma equação com potências de bases diferentes em um membro e outro. 
  


[obm-l] Re: [obm-l] polinômios

2003-01-02 Por tôpico Eder





r(x)= ap(x) + bq(x) = 4x^2 + kx - 8= a(2x^2 - 
3x - 2, ) + b(x^2 - 5x + 1) =
= 
4x^2+kx-8=x^2(2a+b)+x(-3a-5b)+(b-2a)

Pela identidade de polinômios:

2a+b=4 (1)
3a+5b= -k (2)
b-2a=-8 (3)

De (1) e (2) vem que a=3 e b= - 2.Substituindo 
estes valores em (2),tiramos k=1.
Portanto: a+b+k=2.

  - Original Message - 
  From: 
  [EMAIL PROTECTED] 
  
  To: [EMAIL PROTECTED] 
  Sent: Thursday, January 02, 2003 7:16 
  PM
  Subject: [obm-l] polinômios
  Se r(x)= ap(x) + bq(x), 
  com r(x)= 4x^2 + kx - 8, p(x)= 2x^2 - 3x - 2, q(x)=x^2 - 5x + 1, com a, b e k 
  pertencendo aos reais, então a+b+k=? O meu gabarito dá como resposta 
  2. Eu consengui desenvolver apenas o seguinte (não sei se era preciso fazer 
  isso para resolver a questão): 4x^2 + kx - 8= a(2x^2 - 3x - 2, ) + b(x^2 - 5x 
  + 1). A partir disso eu não consegui continuar... 



Re: [obm-l] trigonometria

2003-01-02 Por tôpico Eder



6x/pi = 1 =x=pi/6 =3x=pi/2 =cos(pi/2) = 
0

  - Original Message - 
  From: 
  [EMAIL PROTECTED] 
  
  To: [EMAIL PROTECTED] 
  Sent: Thursday, January 02, 2003 7:15 
  PM
  Subject: [obm-l] trigonometria
  Porque se a tg 
  (9pi/4)=6x/pi, então cos 3x é zero? 



[obm-l] ???

2002-12-30 Por tôpico Eder



Por favor,ajudem-me com essa:


Num triângulo ABC,sejam BD e CE as medidas das bissetrizes dos ângulos ABC 
e BCA,respectivamente.Prove que

ABC é isósceles = BD=CE


Eder



[obm-l] ???

2002-12-23 Por tôpico Eder



Olá colegas de lista,

Sendo a e b ângulos agudos, posso dizer que sen(2a+b)/sen2a = 
sen(2b+a)/sen2b apenas para a=b ?Existem outras possibilidades?
Tava resolvendo um problema proposto aqui lista:

Num triângulo ABC,sejam BD e CE as medidas das bissetrizes dos ângulos ABC 
e BCA,respectivamente.Prove que

ABC é isósceles == BD=CE


Eu estava exatamente na "volta".Através da observação dos ângulos e da lei 
dos senos,chegueià expressão acima.O problema é que não consegui 
desenvolvê-la.Posso concluir logo que a=b apenas olhando a expressão e pelo fato 
de a e b serem agudos (fiz med(ABC)=2a e med(BCA)=2b)?




Re: [obm-l] geometria

2002-12-23 Por tôpico Eder
Trabalhoso esse problema!
Mas vamos lá:

Primeiro vamos trabalhar com o que foi dado,veja que

[AEFD]=3[BCF] = [AEFD]/[BCF]=3 = ( [AEFD]+BCF])/[BCF] )=4 =

= ( [ABCD] - [EBF] - [CDF] )/[BCF] = 4

Pronto,basta calcular cada área separadamente e jogar o resultado na
expressão acima.Chamando a altura do paralelogramo de y e notando a
semelhança dos triângulos EBF e CDF,achamos

[EBF]= x²y/2(x+a)
[CDF]=a²y/2(x+a)

Facilmente,temos que [ABCD]=ay.

Para o cáculo de [BFC],note que [BFC]=[EBC] - [EBF],onde [EBC]=xy/2 e [EBF]
está calculado acima.Assim,
chegamos a [BFC]=axy/2(x+a).

Substituindo na expressão inicial e simplicando,olha só o que apareçe: x² +
2ax -a² = 0 ,donde segue x=a(sqrt2 - 1),
pois x  0.

Muito interessante esse problema.Lembrei-me da dica de um amigo do colégio
que dizia que devemos sempre tentar transformar as expressões dadas no
enunciado do problema,equivalentemente, é claro,de modo a facilitar nosso
trabalho.Esse meu colega resolveu um problema de geometria (que posso postar
depois,assim que encotrá-lo nas minhas coisas!),onde era pedido que se
chegasse a uma relação dada.A expressão inicial não dava pista nenhuma,mas
ele foi dizendo que isso é o mesmo que isso,que, por sua vez,era mesmo que
mais isso,até chegar numa relação boba!De demonstração imediata!

Neste problema,em particular,eu deixei o quadrilátero para de lado.Talvez
alguém da lista possa mostrar que não era tão difícil assim trabalhar com o
quadrilátero,mas eu não vi de cara e preferi trabalhar com os triângulos.


Valeu?!

Eder

- Original Message -
From: Rafael [EMAIL PROTECTED]
To: OBM [EMAIL PROTECTED]
Sent: Thursday, December 19, 2002 3:25 PM
Subject: [obm-l] geometria


 Esse aqui está me dando trabalho:

 Num paralelogramo ABCD,uma reta passando por C
 intercepta a digonal BD em F e o lado AB em E.
 Calcular BE = x, em função de AB = a, sabendo que a
 área do quadrilátero AEFD é o triplo da área do
 triangulo BCF.

 Resposta: x = a.[raiz(2) - 1 ]

 Se alguém tiver uma dica, agradeço...

 Rafael.

 ___
 Busca Yahoo!
 O melhor lugar para encontrar tudo o que você procura na Internet
 http://br.busca.yahoo.com/
 =
 Instruções para entrar na lista, sair da lista e usar a lista em
 http://www.mat.puc-rio.br/~nicolau/olimp/obm-l.html
 O administrador desta lista é [EMAIL PROTECTED]
 =

=
Instruções para entrar na lista, sair da lista e usar a lista em
http://www.mat.puc-rio.br/~nicolau/olimp/obm-l.html
O administrador desta lista é [EMAIL PROTECTED]
=



Re: [obm-l] Re:

2002-12-22 Por tôpico Eder
Oi Paulo,

Acredito que minha tradução estava certa ou pelo menos não comprometia
muito.O que estava errado era o p(p(x))=0 no site do John Scholes...
- Original Message -
From: Paulo Santa Rita [EMAIL PROTECTED]
To: [EMAIL PROTECTED]
Sent: Saturday, December 21, 2002 6:16 PM
Subject: Re: [obm-l] Re:


 Ola Prof Morgado e demais
 colegas desta lista ... OBM-L,

 O contra-exemplo do Prof Morgado, dado abaixo, de forma elegante
 efetivamente encerra a questao. O enunciado esta incorreto. Considerem
agora
 o problema :

 Sejam a, b e c tres reais quaisquer. Se p(x)=ax^2 + bx + c  e
 p(x)=x nao tem raiz real entao p(p(x))=x nao tem raiz real.

 Alias, esta discussao, indiretamente, mostra o quao capciosas podem ser as
 traducoes, nao podendo nunca se resumirem a mera transposicao literal do
 enunciado de um idioma para outro ...

 Este espirito natalino que nos invade, me levou a pensar em Jesus, que os
 cristaos consideram O Cristo Prometido. Depois, por associacao de ideias,
me
 lembrei de um dos Profetas que o antecederam, Salomao. E dai a um dos
 proverbios deste Profeta : Nao respondas ao tolo segundo a sua
estulticia,
 para que nao tambem nao te tornes semelhante a ele

 Um abraco a Todos !
 Paulo Santa Rita
 7,1812,211202






 From: A. C. Morgado [EMAIL PROTECTED]
 Reply-To: [EMAIL PROTECTED]
 To: [EMAIL PROTECTED]
 Subject: Re: [obm-l] Re:
 Date: Sat, 21 Dec 2002 00:30:59 -0200
 
 Vou tentar encerrar a discussao. Tome p(x) = x^2 + 4x + 3. A equaçao p(x)
=
 x reduz-se a  x^2 + 3x + 3 = 0 ue nao tem raiz real pois seu
discriminante
 eh negativo (-3). Como p(-2) = -1, p(p(-2)) = p(-1) = 0, NAO EH VERDADE
que
 p(p(x))=0 nao possua raiz real, pois -2 eh raiz da referida equaçao.
Assim
 como esse, ha muitos contraexemplos que podem ser dados (vejam mensagem
de
 Salvador Addas Zanata).
 Peço desculpas a todos pelo contraexemplo que mandei em mensagens
 anteriores, pois ele estah errado.
 Morgado
 
 Eder wrote:
 
 Esse problema foi retirado do site do John Scholes e o enunciado é:
 
 
 
 Define p(x)=ax²+bx+c.If p(x)=x has no real roots,prove that p(p(x))=0
has
 no real roots.
 
 
 
  - Original Message -
 
  From: A. C. Morgado mailto:[EMAIL PROTECTED]
 
  To: [EMAIL PROTECTED] mailto:[EMAIL PROTECTED]
 
  Sent: Friday, December 20, 2002 5:12 PM
 
  Subject: Re: [obm-l] Re:
 
 
 
 
  Wagner wrote:
 
  Oi pessoal !
 
 
 
  2)Vou supor que a,b,c,x sejam números reais e que a é diferente
  de zero.
 
   Prove que se p(x)=x não tem nenhuma raiz real, então o módulo da
  ordenada do máximo ou do mínimo de f(x)=p(p(x)) é maior que o
  módulo da ordenada do máximo ou do mínimo de g(x)=p(x) -x e
  depois prove que o sinal da derivada de segunda ordem de
  f(x)=p(p(x)) e de g(x)=p(x) -x é o mesmo, assim se a segunda
  função não tem raiz real a primeira também não tem.
 
 
 
  Prova: Primeiro vou provar a segunda hipótese: g '' (x) =2a  ;
   f(x)= a(ax^2 +bx +c)^2 +b(ax^2 +bx +c) +c =
 
  f ' (x) =2a(ax^2 +bx +c)(2ax +b) +b(2ax +b) = f '' (x)
  =4(a^2)(ax^2 +bx +c) +2a(2ax +b)^2 +2ab.
 
  Se a segunda hipótese é verdadeira então f '' (x)/g '' (x)  0
  = 2a(ax^2 +bx +c) +(2ax +b)^2 +b  0 =
 
  2(a^2)(x^2) +2abx +2ac + 4(a^2)(x^2) +4abx +b^2 +b  0  = h(x) =
  6(a^2)(x^2) +6abx +b^2 +2ac +b  0.
 
  Como o coeficiente dominante de h(x) é positivo, devemos apenas
  provar que h(x) não possui raízes reais.
 
  Se h(x) não possui raízes reais então :  36(a^2)(b^2)
  -24{(a^2)(b^2) + 2(a^3)c + (a^2)b}  0 =
 
  12(a^2)(b^2) -48(a^3)c -24(a^2)b  0 = 12b^2 -48ac -24b 0 =
  b^2 -4ac -2b  0 = b^2-4ac  2b ( 1 )
 
 
 
  Para provar ( 1 ) vou fazer algumas considerações:
 
  Devemos ter que p(x)=x não tem raízes reais. Logo (b-1)^2 -4ac 
  0  = b^2 -2b +1 -4ac  0 = b^2 -4ac  2b -1,
 
  logo ( 1 ) é verdadeira se p(x) = x não possui raízes reais CQD.
 
 
 
  Devemos provar agora a primeira hipótese. g ' (x) = 0 = 2ax +b-1
  =0 = x = (1-b)/2a = g ((1-b)/2a) =((b^2-2b+1)/4a) +(-b^2/2a) +c
=
 
  =c +(-b^2-2b+1)/4a = (4ac -b^2-2b+1)/4a =
 
  módulo da ordenada de máximo ou mínimo de g (x) é |
  {-(b^2+2b-1-4ac)/(4a)} | = y
 
  f ' (x) = 2a(ax^2 +bx +c)(2ax+b) +b(2ax +b) = f ' (x) = (2ax
  +b)(2(a^2)(x^2) +2abx +2ac +b)   ; f ' (x) =0 =
 
  (2ax +b) =0 ou (2(a^2)(x^2) +2abx +2ac +b) =0.
 
  O primeiro caso implica em: x= -b/2a
 
  O segundo caso implica em: delta= 4(a^2)(b^2) -4(4(a^3)c +
2(a^2)b).
 
  Vamos provar que delta  0 :  4(a^2)(b^2) -4(4(a^3)c +2(a^2)b) 
  0 = b^2 -4ac -2b  0 = b^2-4ac  2b ( 1 ).
 
  Como ( 1 ) já foi provado, então ficamos só com o caso x= -b/2a =
 
  f(-b/2a) = a((b^2/4a) -(b^2/2a) +c)^2 +b((b^2/4a) -(b^2/2a) +c)
  +c = a(c -(b^2/4a))^2 +b(c -(b^2/4a)) +c =
 
  =a{c^2 -c(b^2)/2a +(b^4/16a^2)}+b(c -(b^2/4a)) +c = a(c^2)
  -c(b^2)/2 +b^4/16a +bc -b^3/4a +c =
 
  módulo da ordenada de máximo ou

Re: [obm-l] Re:

2002-12-20 Por tôpico Eder



Esse problema foi retirado do site do John Scholes 
e o enunciado é:

Define p(x)=ax²+bx+c.If p(x)=x has no real 
roots,prove that p(p(x))=0 has no real roots.


  - Original Message - 
  From: 
  A. C. 
  Morgado 
  To: [EMAIL PROTECTED] 
  Sent: Friday, December 20, 2002 5:12 
  PM
  Subject: Re: [obm-l] Re:
  Wagner wrote:
  




Oi pessoal !

2)Vou supor que a,b,c,x sejam números reais e 
que a é diferente de zero.
Prove que se p(x)=x não tem nenhuma raiz 
real, entãoo módulo da ordenada do máximo ou do mínimode 
f(x)=p(p(x)) é maior que o módulo da ordenada do máximo ou do mínimo de 
g(x)=p(x) -x e depois prove queo sinal da derivada de segunda ordem de 
f(x)=p(p(x)) e de g(x)=p(x) -x é o mesmo, assim se a segunda função não tem 
raiz real a primeira também não tem.

Prova: Primeiro vou provar a segunda 
hipótese:g '' (x) =2a ; f(x)= a(ax^2 +bx +c)^2 +b(ax^2 +bx 
+c) +c = 
f ' (x) =2a(ax^2 +bx +c)(2ax +b) +b(2ax +b) 
= f '' (x) =4(a^2)(ax^2 +bx +c) +2a(2ax +b)^2 +2ab.
Se a segunda hipótese é verdadeira então f '' 
(x)/g '' (x)  0 =2a(ax^2 +bx +c) +(2ax +b)^2 +b  0 = 

2(a^2)(x^2) +2abx +2ac + 4(a^2)(x^2) +4abx +b^2 
+b  0 = h(x) = 6(a^2)(x^2) +6abx +b^2 +2ac +b 
0.
Como o coeficiente dominante de h(x) é 
positivo, devemos apenas provar que h(x) não possui raízes 
reais.
Se h(x) não possui raízes reais então : 
36(a^2)(b^2) -24{(a^2)(b^2) + 2(a^3)c +(a^2)b}  0 
=
12(a^2)(b^2) -48(a^3)c -24(a^2)b  0 = 
12b^2 -48ac -24b 0 = b^2 -4ac -2b  0 = b^2-4ac  2b ( 1 
)

Para provar ( 1 ) vou fazer algumas 
considerações:
Devemos ter que p(x)=x não tem raízes reais. 
Logo (b-1)^2 -4ac  0 = b^2 -2b +1 -4ac  0 = b^2 
-4ac 2b -1,
logo ( 1 ) é verdadeira se p(x) = x não possui 
raízes reais CQD.

Devemos provar agora a primeira hipótese. g ' 
(x) = 0 = 2ax +b-1 =0 = x = (1-b)/2a = g ((1-b)/2a) 
=((b^2-2b+1)/4a) +(-b^2/2a) +c =
=c +(-b^2-2b+1)/4a = (4ac -b^2-2b+1)/4a 
=

módulo da ordenada de máximo ou mínimo de g (x) 
é | {-(b^2+2b-1-4ac)/(4a)} | = y
f ' (x) = 2a(ax^2 +bx +c)(2ax+b) +b(2ax 
+b)= f ' (x) = (2ax +b)(2(a^2)(x^2) +2abx +2ac 
+b);f ' (x) =0 =
(2ax +b) =0 ou (2(a^2)(x^2) +2abx +2ac +b) 
=0.
O primeiro caso implica em: x= 
-b/2a
O segundo caso implica em: delta= 4(a^2)(b^2) 
-4(4(a^3)c + 2(a^2)b).
Vamos provar quedelta  0: 
4(a^2)(b^2) -4(4(a^3)c +2(a^2)b)  0 = b^2 -4ac -2b  0 = 
b^2-4ac  2b ( 1 ).
Como ( 1 ) já foi provado, então ficamos só com 
o caso x= -b/2a =
f(-b/2a) = a((b^2/4a) -(b^2/2a) +c)^2 
+b((b^2/4a) -(b^2/2a) +c) +c = a(c -(b^2/4a))^2 +b(c -(b^2/4a)) +c 
=
=a{c^2 -c(b^2)/2a +(b^4/16a^2)}+b(c -(b^2/4a)) 
+c = a(c^2) -c(b^2)/2 +b^4/16a +bc -b^3/4a +c =
módulo da ordenada de máximo ou mínimo de f (x) 
é | {a(c^2) -c(b^2)/2 +b^4/16a +bc -b^3/4a +c} | = z.
Como a segunda hipótese é verdadeira então se 
g(x) tem máximo definido f(x) também tem, e se g(x)
tem mínimo definido f(x) também tem. Temos que 
se p(x) =x não tem raiz real f '(x) e g'(x) só tem uma
raiz real, note que se a  0, g(x) tem 
mínimo e se a  0, g(x) tem máximo. Logo para provar a primeira hipótese, 
temos
que considerar 2 casos : a  0 e a  
0.
Suponha que a primeira hipótese seja 
falsa:
a  0 = y  zey,z  
0= g((1-b)/2a)  f(-b/2a) =-b^2/4a -b/2a +1/4a 
+c a(c^2) -c(b^2)/2 +b^4/16a +bc -b^3/4a +c =
-4b^2 -8b +4  16(a^2)(c^2) -8ac(b^2) +b^4 
+16bc -4b^3 = 16(a^2)(c^2) -8ac(b^2) +b^4 +16bc -4b^3 +4b^2 -8b +4 =h(a) 
 0
Considere ( 2 ) uma função do 2º grau de 
variável a. Temos a  0, logo:
64(b^4)(c^2) -64(b^4)(c^2) -64(c^2)(16bc -4b^3 
+4b^2 -8b +4)  0 = 16bc -4b^3 +4b^2 -8b +4 0 ( 3 ).
De ( 2 ) vem que: (b^2 -4ac)^2 -(16bc 
-4b^3 +4b^2 -8b +4)  0 . Absurdo !
Para o caso a  0 = y  z, temos um 
raciocínio análogo, provamos que se a  0, então h(a)  0, logo o 
delta de h(a)
é negativo, o que nos leva a conclusão de que 
(b^2 -4ac)^2  0 Absurdo !
Logo a primeira hipótese é verdadeira, porque é 
absurdo que ela seja falsa se a segunda hipótese é verdadeira,
Logo p(x)=x não ter raízes reais implica na 
segunda hipótese qua implica na primeira.
Se a primeira e a segunda hipóteses são ambas 
verdadeiras, isso implica que p(p(x))=0 não tem nenhuma raiz 
real
CQD.
Isso eh falso. Se p(x) = x^2 +3x+2, a equaçao p(p(x))=0 
tem uma raiz real entre -1 e 0.
OBS:Me desculpem pelo e-mail que eu mandei sem 
querer antes, ele estava com a resposta pela metade.


André T.


  - 
  Original Message - 
  From: 
  Eder 
  To: 
  [EMAIL PROTECTED] 
  Sent: 
  Thursday, December 19, 2002 5:32 PM
  
  Gostaria da ajuda de vcs nestes problemas

[no subject]

2002-12-19 Por tôpico Eder



Gostaria da ajuda de vcs nestes problemas 
russos:


1)Um triângulotem área 1 e lados a  = b 
 = c.Prove que b²  = 2.

2)Defina p(x)=ax²+bx+c.Se p(x)=x não tem nenhuma 
raiz real, prove que p(p(x)) = 0 também não tem nenhuma raiz real.

Grato pela ajuda.

Eder


Re: [obm-l] Ajuda Sobre um site (estranho!!)

2002-12-19 Por tôpico Eder



Puxa,um site brasileiro com tudo isso seria o 
máximo!

  - Original Message - 
  From: 
  Johann Peter Gustav Lejeune 
  Dirichlet 
  To: [EMAIL PROTECTED] 
  Sent: Thursday, December 19, 2002 7:02 
  PM
  Subject: Re: [obm-l] Ajuda Sobre um site 
  (estranho!!)
  
  Acho que ninguem entendeu:eu tenho provas traduzidas e queria uma ajuda 
  para coloca-las em rede do jeito mais adequado.Para isso eu precisaria de 
  alguem que soubesse eascrever em algo mais compacto que o Word(alias o Windows 
  ja esta ficando jurassico,digo,cambriano :) ),como o .ps,ou .pdf ,e tambem o 
  .tex e .dvi.E claro uma caixade e-mails onde eu e os webmasters 
  estariamos vendo e-mails com soluçoes de problemas nao-resolvidos e novas 
  soluçoes mais elegantes ou nao de problemas ja respondidos. 
  E ainda,como eu faço coleçoes de problemas respondidos por mim e por 
  varios professores que conheço,seria legal eu fazer um link com varias dessas 
  perolas,desde a demonstraçao do Postulado de Bertrand ate a demonstraçao 
  elementar do TNP.Elas serviriam como treinamento para olimpiadas em geral. 
  Entenderam? 
  Ass.:Johann 
  Eduardo Casagrande Stabel [EMAIL PROTECTED] 
  wrote: 
  Caro 
Anderson Torres e Marcelo Leitner,muito mais interessante do que 
fazer um site contendo as provas que contémoutros sites (com o do John 
Scholes) é fazer um site que contém as provas*traduzidas* para o 
Português. Então acho que o trabalho verdadeiro é o detradução. Acredito 
que montar a página seja a tarefa mais simples.Coincidentemente, 
este trabalho de tradução já está sendo desenvolvido. OPaulo Santa Rita 
possui muitas questões russas (por exemplo), e eu e eletemos trabalhado 
na tradução de problemas de nível universitário, porenquanto. A 
olimpíada Putnam, que está no site do Scholes, já foipraticamente toda 
traduzida por nós: são mais de 800 problemas. Porenquanto, ainda há 
muitos erros de tradução, dúvidas quanto ao significadode questões, e 
coisas do tipo. Se você quiser dar uma olhada, 
visitewww.geocities.com/olimpiadag a! ucha/putnam.html,onde 
eu disponibilizei temporariamente algumas delas. Eu e o Paulo iríamosnos 
empenhar em dar um acabamento final, decidir quais 
formatosdisponibilizar, e fazer um sistema de busca, identificando por 
assuntos cadaquestões. Só que iríamos fazer isso em Janeiro (quando 
começam as férias) esó DEPOIS anunciaríamos o trabalho 
pronto.Recentemente, eu já comecei a traduzir as IMO's, traduzi 
muito pouca coisa,só as 5 primeiras. Seria legal - se você(s) estiverem 
de acordo em trabalharseriamente conosco - dividir tarefas e trabalho de 
tradução para que duaspessoas não façam o mesmo serviço repetidas 
vezes.Não sei se o Paulo vai se incomodar com o fato de eu ter 
avisado do trabalhosem ele estar pronto... mas achei que seria uma boa 
idéia unirmos forças emum mesmo sentido, do que ficarmos latindo 
separadamente um para cada lado...Abraço,Eduardo.From: 
"Marcelo Leitner" <[EMAIL PROTECTED]>!  On Thu, Dec 12, 2002 at 
04:37:29PM -0300, Johann Peter Gustav LejeuneDirichlet wrote: 
 Ola turma da Lista OBM   Estou pensando em 
montar um site com provas das varias Olimpiadas mundoafora,mais ou menos 
como o John Scholes fez.No momento tenho provas 
daCone-Sul(algumas),Iberoamericana,IMO,Chinesa,Estadunidense,APMO,Balcanica,Rioplatense(algumas),listas 
de treinamento e seleçao para provas de variasolimpiadas,e milhares de 
problemas legais(de Geometria principalmente).Ate aVingança Olimpica vai 
entrar na festa!!!   Mas primeiro preciso de algumas 
coisas:um segundo webmaster(nao voupoder fazer varias coisas sozinho) e 
um provedor decente que de o espaço deque preciso(afinal tenho prova a 
dar com o pau!) e leitores curtos tipo TeXe GSView.Se alguem puder me 
ajudar agradeço muito!   Ass.:Johann 
---end quoted text--- Ae Johann ,! eu me disponibilizo se 
quiser minha ajuda. Tenho conhecimentosem html, php, mysql e 
LaTeX. Quanto ao provedor, eu tenho adsl, a banda de upload eh 1/2 
pequena,150kbits/s, mas eh 24hrs (com direito a nobreak :)) e 
jah tenho um servidor httpaberto.. o unico problema eh que a 
porta 80 eh firewalled pela telepar, aihcomplica p/ ter um 
dominio tipo www.algo.com.br, mas se espaco p/ as provas 
forprblm, dah p/ deixar os .htm* .php em algum provedor e as 
provas em si aqui.. bom, era isso, qlquer coisa meu email tah aih em 
baixo :) ah sim, o server roda em ambiente linux, com servidor ssh 
aberto e talz.. []'s -- Marcelo R Leitner 
<[EMAIL PROTECTED]> 
= 
Instruções para entrar na lista, sair da lista e usar a lista em 
http://www.mat.puc-rio.br/~nicolau/olimp/obm-l.html O! administrador 
desta lista é <[EMAIL PROTECTED]> 

[obm-l] probabilidade

2002-12-17 Por tôpico Eder



Por favor,ajudem-me com esta:


Em um balcão de supermercado, foram esquecidas duas 
sacolas.Uma continha 3 latas de atum,2 de ervilha e 5 de sardinha;a 
outra,x latas de atum,3 de ervilha e 3 de sardinha.Escolhe-se 
ao acaso uma sacola e retira-se uma lata.Qual é o menor valor de x 
para que a probabilidade de tratar-se de uma lata de atum seja,no 
mínimo, 50%?


Eder


[obm-l] Re: [obm-l] dúvida polinômios

2002-12-01 Por tôpico Eder
Muito interessante.Valeu,Fábio!


- Original Message -
From: joao dias [EMAIL PROTECTED]
To: [EMAIL PROTECTED]
Sent: Saturday, November 30, 2002 8:40 PM
Subject: Re: [obm-l] dúvida polinômios


 At 18:06 29/11/2002 -0200, you wrote:
 Olá companheiros de lista,
 
 Eu gostaria de saber o que há de especial quando temos p(x) = p(x-k) ou
 p(x)=p(k-x),k real,para um polinômio qualquer.Tipo,que informações
 interessantes podem ser retiradas de uma igualdade dessas,como essas
 igualdades podem ser úteis na resolução de problemas...
 
 Agradeço por quaisquer comentários.
 
 
 Eder

 Se p(x) = p(x-k) então ... = p(n-2k) = p(n-k) = p(n) = p(n+k) = p(n+2k) =
 ... Assim, q(x) = p(x) - p(n) tem infitas raízes, logo é identicamente
nulo
 == p(x) é constante.

 Se p(x) = p(k-x), então p(x+k/2) = p(-x+k/2) == q(x) = p(x-k/2) é um
 polinômio par (q(x) = q(-x), o que implica que todos os seus termos têm
 expoentes pares).

 []s,

 Fábio Dias (pelo email de meu pai)

 =
 Instruções para entrar na lista, sair da lista e usar a lista em
 http://www.mat.puc-rio.br/~nicolau/olimp/obm-l.html
 O administrador desta lista é [EMAIL PROTECTED]
 =

=
Instruções para entrar na lista, sair da lista e usar a lista em
http://www.mat.puc-rio.br/~nicolau/olimp/obm-l.html
O administrador desta lista é [EMAIL PROTECTED]
=



[obm-l] dúvida polinômios

2002-11-29 Por tôpico Eder



Olá companheiros de lista,

Eu gostaria de saber o que há de especial quando temos p(x) = 
p(x-k) ou p(x)=p(k-x),k real,para um polinômio qualquer.Tipo,que informações 
interessantes podem ser retiradas de uma igualdade dessas,como essas igualdades 
podem ser úteis na resolução de problemas...

Agradeço por quaisquer comentários.


Eder





Re: [obm-l] da Vunesp

2002-11-28 Por tôpico Eder
Sendo f uma função injetora,os itens b,e podem ser eliminados.Pelos dados da
questão,não se pode afirmar com certeza o que consta nos itens a,c.Resta o
item d.De fato,se f for sobrejetora,para todo a pertencente a B,existe x
pertencente a A tal que a=f(x).

Acho que é isso...



- Original Message -
From: pichurin [EMAIL PROTECTED]
To: [EMAIL PROTECTED]
Sent: Thursday, November 28, 2002 1:03 AM
Subject: [obm-l] da Vunesp


 Sejam A e B dois conjuntos não vazios  tais que
 a sua intersecção é o conjunto vazio e seja f de A em
 B uma função injetora.Se a é um elemento de B então,
 para x pertencente a A, a equação f(x)=a
 a) Não tem solução
 b)tem duas soluções
 c) tem umaúnica solução
 d) terá solução se a função f for sobrejetora
 e) tem mais que duas soluções.



 ___
 Yahoo! Acesso Grátis
 Internet rápida, grátis e fácil. Faça o download do discador agora mesmo.
 http://br.acesso.yahoo.com/
 =
 Instruções para entrar na lista, sair da lista e usar a lista em
 http://www.mat.puc-rio.br/~nicolau/olimp/obm-l.html
 O administrador desta lista é [EMAIL PROTECTED]
 =

=
Instruções para entrar na lista, sair da lista e usar a lista em
http://www.mat.puc-rio.br/~nicolau/olimp/obm-l.html
O administrador desta lista é [EMAIL PROTECTED]
=



Re: [obm-l] Mais ajuda!

2002-11-20 Por tôpico Eder



3)(1 + i)^n = (1- i )^n == (1+i/1-i)^n=1 == 
(1+i)^(2n)=2^n
Mas 1+i=sqrt2cis(pi/4) ==(1+i)²=2cis(pi/2) 
==(1+i)^(2n)=2^n cis(npi/2)
Devemos ter,então,cis(npi/2)=1=cis0 == npi/2=0+2kpi== 
n=4k ; k inteiro

Acho que é isso...
- Original Message - 

  From: 
  Sharon 
  Guedes 
  To: [EMAIL PROTECTED] 
  Sent: Wednesday, November 20, 2002 12:54 
  AM
  Subject: [obm-l] Mais ajuda!
  
  
  Olá pessoal, será que alguém poderia 
  me ajudar nessas questões:
  1) Determine o conjunto solução da 
  equação ÷ 
  z÷ ² + z  z .`z = 3 + 3i 
  
   
  Resposta : 3 + 3i
  2)Sabendo que z é um número complexo tal quez . `z= 24 ,calcule o módulo de 
  z.
  Resposta: 2Ö 6
   
  
   
  (UFRGS) A igualdade (1 + i)^n = (1- i )^n se verifica se e 
  somente se:
  
n = 4K, k Î z 
n = 0 
n é ímpar 
n é par 
n é primo.
  At. Sharon. 
  
  
  Yahoo! GeoCitiesTudo 
  para criar o seu site: ferramentas fáceis de usar, espaço de sobra e 
  acessórios.


[obm-l] ???

2002-11-19 Por tôpico Eder



Olá,

Eu gostaria de passar um probleminha que não vi 
ainda como resolver:


Dois colecionadores de selos têm,juntos,500 
selos.Cada colecionador comprou um álbum para colocar seus selos.Os dois álbuns 
eram idênticos,tendo o mesmo número de páginas.

Se o primeiro colecionador colocar exatamente 21 
selos em cada página,ele vai conseguir colocar todos os seus selos e usar todas 
as páginas do álbum.

Se o segundo colecionador colocar 20 de seus selos 
em cada página do álbum,sobrarão alguns selos.Caso ele coloque 23 selos em cada 
página,sobra pelo menos uma,totalmente vazia,podendo haver ainda uma outra 
página com menos de 20 selos.

Quantas páginas há no 
álbum???


[obm-l] Re: [obm-l] ângulo?

2002-11-14 Por tôpico Eder
Seja O o centro da circunferência.Ligando o centro O aos pontos B  e C,será
formado o triângulo equilátero BOC.Basta notar agora que  ângulo(BOC)= 60º =
2.ângulo(BAC).Logo, ângulo(BAC)=30º.
- Original Message -
From: Juliana Löff [EMAIL PROTECTED]
To: [EMAIL PROTECTED]
Sent: Thursday, November 14, 2002 8:58 PM
Subject: [obm-l] ângulo?


 E aí, pessoal?
 Agradeço uma ajuda aqui nessa questão!
 Ju
 ---
 Numa circunferência está inscrito um triângulo ABC; seu lado BC é igual ao
 raio da circunferência. O ângulo BAC mede:

 a) 15º
 b) 30º
 c) 36º
 d) 45º
 e) 60º

 =
 Instruções para entrar na lista, sair da lista e usar a lista em
 http://www.mat.puc-rio.br/~nicolau/olimp/obm-l.html
 O administrador desta lista é [EMAIL PROTECTED]
 =

=
Instruções para entrar na lista, sair da lista e usar a lista em
http://www.mat.puc-rio.br/~nicolau/olimp/obm-l.html
O administrador desta lista é [EMAIL PROTECTED]
=



Re: [obm-l] IME 2003

2002-11-10 Por tôpico Eder



x=raiz cúbica(20+14sqrt2)+raiz 
cúbica(20-14sqrt2)

Eleve ao cubo os dois lados,usando a 
identidade:

(A+B)³=A³+B³+3AB(A+B)

Fica:

x³=20+14sqrt2+20-14sqrt2+3.raiz 
cúbica8.x (note que 
A+B=x)

x³-6x-40=0

É fácil ver que 4 é raiz dessa equação,mostrando 
que a expressão inicial é um inteiro múltiplo de 4.
Quanto a sua outra pergunta,eu não saberia te dar 
uma regra geral...Sei lá,depende do problema...Vamos ver se alguém da lista dá 
alguma dica.

  - Original Message - 
  From: 
  Wander 
  Junior 
  To: [EMAIL PROTECTED] 
  Sent: Sunday, November 10, 2002 10:47 
  AM
  Subject: [obm-l] IME 2003
  
  Esta questão é da prova do IME que foi realizada 
  nesta semana que passou. Alguém poderia me dar uma ajuda.
  
  
  Qual a melhor forma de resolver exercícios em que 
  se tem que demonstrar ou provar as coisas, tipo essas questões do IME 
  ?
  
  Obrigado pela ajuda.
  Wander


[obm-l] ???

2002-11-09 Por tôpico Eder



1)Mostrar que n!.n!  n^n 
,n2.
2)Determinar todos os inteiros positivos x e y tais 
que x³-y³=xy-61.

Obrigado por quaisquer comentários.


Eder


[obm-l] teoria dos números

2002-11-04 Por tôpico Eder



 
Gostaria de ajuda nestes problemas:

1)Se 2^k - 1,onde k é um inteiro maior que 2,é 
primo,prove que k é primo.
2)Mostre que ^() + ^() é divisível 
por 7.
3)Prove que se um dos números 2^n - 1 e 2^n + 1 é 
primo,então óutro é composto.




[obm-l] Re: [obm-l] teoria dos números

2002-11-04 Por tôpico Eder



Valeu pela ajuda,Morgado.

  - Original Message - 
  From: 
  Augusto 
  César Morgado 
  To: [EMAIL PROTECTED] 
  Sent: Monday, November 04, 2002 7:09 
  PM
  Subject: Re: [obm-l] teoria dos 
  números
  3) 2^n-1, 2^n, 2^n+1 sao tres inteiros consecutivos; um deles 
  eh multiplo de 3...1) x^n - 1 = (x - 1) 
  [x^(n-1)+x^(n-2)+...+1]Logo, se x eh inteiro, x^n - 1 eh divisivel por x - 
  1Se k eh composto, k = ab com a e b inteiros maiores que 1.2^(ab)-1 = 
  x^b -1 com x = 2^a eh divisivel por 2^a - 1.. .Eder 
  wrote:
  007e01c28435$d3a2f340$3c02fea9@Eder" type="cite">



 
Gostaria de ajuda nestes 
problemas:

1)Se 2^k - 1,onde k é um inteiro maior que 2,é 
primo,prove que k é primo.
2)Mostre que ^() + ^() é 
divisível por 7.
3)Prove que se um dos números 2^n - 1 e 2^n + 1 
é primo,então óutro é composto.




Re: [obm-l] desafio !

2002-11-03 Por tôpico Eder



Uma vez alguém me falou de uma analogia 
interessante que poderia ser utilizada neste problema...
É o seguinte:

Consideremos um polígono convexo de 11 lados e,é 
claro,de 11 vértices.Você posicionaria 1homem em cada vértice,assim estes 
estariam em exatamente duas patrulhas e cada duas patrulhas teriam um homem em 
comum.Agora, considere as diagonais.Imaginando uma diagonal como um homem,vemos 
que aqui também é respeitado o fato de que cada homem estaria em exatamente duas 
patrulhas e cada duas patrulhas,ligadas pela diagonal,teriam um homem em 
comum.Assim o total de homens é:

nº vértices+nº diagonais= 11 +11(11-3)/2 = 
55.


Seja n o número de homens por patrulha.Temos que 
11*n=2*55 (pois cada homem foi contado duas vezes),daí n=10.

Espero que esteja certo.


Eder


  - Original Message - 
  From: 
  Wander 
  Junior 
  To: [EMAIL PROTECTED] 
  Sent: Sunday, November 03, 2002 2:09 
  PM
  Subject: [obm-l] desafio !
  
  Dúvida:
  
  Um comandante de companhia convocou voluntários 
  para a constituição de 11 patrulhas. Todas elas são formadas pelo mesmo número 
  de homens. Cada homem participa de exatamente duas patrulhas. Cada duas 
  patrulhas tem somente um homem em comum. Determine o múmero de voluntários e 
  integrantes de uma patrulha.
  
  
  Agradeço desde já.
  Wander


[obm-l] Re: [obm-l] questão IME

2002-11-03 Por tôpico Eder




(2a+1)+(2a+3)+(2a+5)+...(2a+2n-1)=7³
2na+(1+3+5+...+2n-1)=7³
2na+n(1+2n-1)/2=7³
2na+n²=7³
n(n+2a)=7³

Observe que n e a são inteiros,em 
particular,n0.Agora temos as possibilidades:

1)n=1 e n+2a=7³ == 2a=7³-1 
==2a=342

Nesse caso, temos um único termo 
(2a+1)=343.

2)n=7 e n+2a=7² == 2a=7²-7=42

Nesse caso,os termos são 43,45,47,49,51,53 
e 55 (7 termos).

3)n=7² e n+2a=7 ==2a= -42

Nesse caso,os termos são 
-42,--40,-38,...,54 (7² 
termos)

4)n=7³ e n+2a=1 == 2a= -342

Nesse caso, os termos são 
-341,-340,...,343 (7³ termos)


Bom,acho que é isso.


Eder

  - Original Message - 
  From: 
  Wander 
  Junior 
  To: [EMAIL PROTECTED] 
  Sent: Sunday, November 03, 2002 2:41 
  PM
  Subject: [obm-l] questão IME
  
  Alguem pode me ajudar com esta questão do IME do 
  ano de 1997-1998?
  
  Uma soma finita de números inteiros consecutivos, 
  ímpares, positivos ou negativos, é igual a 7^3 (7 elevado ao 
  cubo).
  Determine os termos desta soma.
  
  
  Obrigado.


[obm-l] ??

2002-10-30 Por tôpico Eder



Olá,

Não tô conseguindo resolver isto (em R) :

x+sqrt(x²-10x+9)  sqrt(x+2sqrt(x²-10x+9) )

Ah!E como eu provaria que não existe uma função real f tal que 
f(f(x))=x²-1996?

Grato por quaisquer comentários.

Eder


[obm-l] Re: [obm-l] QUESTÃO RIDÍCULA, PORÉM COMPLICADA !!

2002-10-26 Por tôpico Eder



Se duas das raízes são a e -a,então a soma das três 
raízes dará a terceira.Temos:

a+(-a)+b= -k/8 == b = -k/8 (terceira 
raiz)

Substituindo tal valor de b na equação 
original:

8*(-k³/8³)+(k²/8²)k-18(k/8)+9 = 0 == 9-18k/8=0 
== 1-k/4=0 == k=4

Espero que esteja tudo direitinho...



  - Original Message - 
  From: 
  Afemano 

  To: [EMAIL PROTECTED] 
  Sent: Saturday, October 26, 2002 5:46 
  PM
  Subject: [obm-l] QUESTÃO RIDÍCULA, PORÉM 
  COMPLICADA !!
  
  Alguém me da uma luz nessa questão plz 
  ?!?!?
  
  Seja P(x) = 8x^3 + kx^2 - 18x + 9.
  E suas raízes "a" e "-a", determine 
  "k".
  
  Eu tentei por Giraldi e chegueia conclusão 
  de que k  0 mas. não achei em lugar nenhum com isso..
  
  Gabriel
  


[obm-l] equação

2002-10-24 Por tôpico Eder



Olá,

Gostaria de uma ajudinha na equação 
abaixo:

4senx + 2cosx - 3tgx - 2=0

Já tentei uma monte de coisa aqui e 
nada...


[obm-l] ??

2002-10-19 Por tôpico Eder



Achar as soluções inteiras de 
1/x+1/y=1/1998.


[obm-l] Re: [obm-l] Re: [obm-l] Re: [obm-l] dúvida

2002-10-01 Por tôpico Eder

Seguindo o racionínio do Villard,veja que se representarmos na base 2 o
valor dos pontos ganhos em cada pergunta ,teremos sempre representações
diferentes.Teremos sempre um 1 e alguns zeros,ou apenas zeros (no caso de
repostas erradas).Nunca teremos duas pontuações com 1's na mesma
posição.Exemplos hipotéticos de pontuações:


0010 0100 0001,mas não aparecem 0110 ou 0010 novamente
O que isso tem de especial?

Na hora de somarmos as pontuações,os 1's aparecerão  na ordem das perguntas
respondidas corretamente!De fato,basta lembrar como se soma números
binários,como teremos sempre 1 em cima de zero ou zero em cima de
zero

Pois é,basta representar o resultado na base 2 e contar da direita para
esquerda,essa é a ordem das perguntas.Daí ele ter respondido as perguntas
2,6,7 e 10.Note que a representação na base 2 não compromete,ou melhor, ele
está por trás de tudo!Caso a pergunta 1 tivesse sido respondida corretamente
o candidato ganharia 1*2º=1 ponto,conforme expresso no enunciado.Espero ter
ajudado.

Eder

- Original Message -
From: Henrique Branco [EMAIL PROTECTED]
To: [EMAIL PROTECTED]
Sent: Monday, September 30, 2002 11:59 PM
Subject: [obm-l] Re: [obm-l] Re: [obm-l] dúvida


 Pergunta besta... Mas se 610 = (1001100010)_2, por que ele acertou as
 perguntas 2, 6, 7, 10?
 Desculpem pelo nivel primario da pergunta... Mas...
 Grato,
 Henrique.

 - Original Message -
 From: Rodrigo Villard Milet
 To: [EMAIL PROTECTED]
 Sent: Saturday, September 28, 2002 9:00 PM
 Subject: [obm-l] Re: [obm-l] dúvida

 Escreva 610 na base 2 : 610 = (1001100010)_2. Como sabemos que a
 representação na base 2 é única, ele acertou as perguntas 2,6,7 e 10.
 Villard


 -Mensagem original-
 De: Mário Pereira [EMAIL PROTECTED]
 Para: [EMAIL PROTECTED] [EMAIL PROTECTED]
 Data: Sábado, 28 de Setembro de 2002 11:22
 Assunto: [obm-l] dúvida

 Olá, se alguém puder, me dê uma dica:

 Em um jogo de televisão, um candidato deve responder a 10 perguntas. A
 primeira vale 1 ponto, a segunda vale 2 pontos, e assim, sucessivamente,
 dobrando sempre. O candidato responde a todas as perguntas e ganha os
pontos
 correspondentes às respostas que acertou, mesmo que erre algumas. Se o
 candidato obteve 610 pontos, quantas perguntas acertou?

 Obrigado,

 Mário.


 =
 Instruções para entrar na lista, sair da lista e usar a lista em
 http://www.mat.puc-rio.br/~nicolau/olimp/obm-l.html
 O administrador desta lista é [EMAIL PROTECTED]
 =

=
Instruções para entrar na lista, sair da lista e usar a lista em
http://www.mat.puc-rio.br/~nicolau/olimp/obm-l.html
O administrador desta lista é [EMAIL PROTECTED]
=



[obm-l] Mais uma...

2002-09-30 Por tôpico Eder



Essa foi da olimpíada russa:


"Sabendo que a quadrática x²+ax+b+1 tem raízes 
inteiras e positivas,mostre que a²+b² é composto."

Bom,supondo X1 e X2 raízes,analisei as 
possibilidades de serem ambas ímpares,uma par e outra ímpar e as duas pares.O 
único problema que encontrei foi para o último caso.X1 e X2 pares implicab 
ímpar e a par.Diferente dos outros casos,não pude ou não vi como concluir que 
a²+b² é composto.Aguardo comentários.


[obm-l] ???

2002-09-29 Por tôpico Eder



Olá,



Gostaria de ajuda na situação abaixo.


Sendo a,b e c inteiros positivos,resolva: 
(1+1/a)(1+1/b)(1+1/c)=2.

Eu estava tentando e não me pareceu haver 
solução...


[obm-l] analítica

2002-09-27 Por tôpico Eder



Olá,

Dada a equação de uma circunferência : 
(x-4)²+(y-3)²=9,como faço para achar o ponto sobre a mesma que está a menor 
distância da origem?Alguém poderia dar uma dica?







Re: [obm-l] ???

2002-09-07 Por tôpico Eder



Vou checar se não distorci o problema.Ele caiu na 
olimpíada interna do meu colégio.

  - Original Message - 
  From: 
  Augusto 
  César Morgado 
  To: [EMAIL PROTECTED] 
  Sent: Saturday, September 07, 2002 9:07 
  PM
  Subject: Re: [obm-l] ???
  Ha alguma coisa errada no problema 2.Em um polinomio de 
  coeficientes inteiros, P(a) - P(b) eh divisivel por a-b (a, b inteiros, 
  naturalmente).Entao, 247 - 17 = 230 deveria ser divisivel por 
  32 - 21 = 11.Eder wrote:
  



Gostaria de ajuda nestes 
problemas:

1)Encontre todas as soluções reais de 
cosx+(cosx)^5+cos7x=3.

Bom,eu tranformei cosx+cos7x em produto,depois 
saí fazendo simplificações para ficar somente com cosx.Substituindo cosx po 
m,cheguei a um polinômio bem "estranho".Foi fácil checar que m=1 
==cosx=1==x=2kpi é solução (na verdade é fácil de ver isso no 
enunciado),porém não soube mais o que fazer depois disso,para ver se há 
outras soluções.

2)Considere um polinômio de coeficientes 
inteiros.Sabe-se que p(21)=17,p(32)=-247 e p(37)=33.Prove que se para 
algum N tivermos p(N)=51.Então N=26.


Obrigado pela ajuda.
    
Eder




Re: [obm-l] ???

2002-09-07 Por tôpico Eder



Agora que percebi! Não dá para ver direito que 
p(32)= - 247.Assim, -247-17=264 que é divisível por 11.Continuo esperando alguma 
luz nesse problema...

  - Original Message - 
  From: 
  Augusto 
  César Morgado 
  To: [EMAIL PROTECTED] 
  Sent: Saturday, September 07, 2002 9:07 
  PM
  Subject: Re: [obm-l] ???
  Ha alguma coisa errada no problema 2.Em um polinomio de 
  coeficientes inteiros, P(a) - P(b) eh divisivel por a-b (a, b inteiros, 
  naturalmente).Entao, 247 - 17 = 230 deveria ser divisivel por 
  32 - 21 = 11.Eder wrote:
  



Gostaria de ajuda nestes 
problemas:

1)Encontre todas as soluções reais de 
cosx+(cosx)^5+cos7x=3.

Bom,eu tranformei cosx+cos7x em produto,depois 
saí fazendo simplificações para ficar somente com cosx.Substituindo cosx po 
m,cheguei a um polinômio bem "estranho".Foi fácil checar que m=1 
==cosx=1==x=2kpi é solução (na verdade é fácil de ver isso no 
enunciado),porém não soube mais o que fazer depois disso,para ver se há 
outras soluções.

2)Considere um polinômio de coeficientes 
inteiros.Sabe-se que p(21)=17,p(32)=-247 e p(37)=33.Prove que se para 
algum N tivermos p(N)=51.Então N=26.


Obrigado pela ajuda.
    
Eder




Re: [obm-l] ???

2002-09-07 Por tôpico Eder



-247-17 = -264

  - Original Message - 
  From: 
  Eder 
  To: [EMAIL PROTECTED] 
  Sent: Saturday, September 07, 2002 10:07 
  PM
  Subject: Re: [obm-l] ???
  
  Agora que percebi! Não dá para ver direito que 
  p(32)= - 247.Assim, -247-17=264 que é divisível por 11.Continuo esperando 
  alguma luz nesse problema...
  
- Original Message - 
From: 
Augusto César Morgado 
To: [EMAIL PROTECTED] 
Sent: Saturday, September 07, 2002 9:07 
PM
Subject: Re: [obm-l] ???
Ha alguma coisa errada no problema 2.Em um polinomio de 
coeficientes inteiros, P(a) - P(b) eh divisivel por a-b (a, b inteiros, 
naturalmente).Entao, 247 - 17 = 230 deveria ser divisivel por 
32 - 21 = 11.Eder wrote:

  
  

  Gostaria de ajuda nestes 
  problemas:
  
  1)Encontre todas as soluções reais de 
  cosx+(cosx)^5+cos7x=3.
  
  Bom,eu tranformei cosx+cos7x em 
  produto,depois saí fazendo simplificações para ficar somente com 
  cosx.Substituindo cosx po m,cheguei a um polinômio bem "estranho".Foi 
  fácil checar que m=1 ==cosx=1==x=2kpi é solução (na verdade 
  é fácil de ver isso no enunciado),porém não soube mais o que fazer depois 
  disso,para ver se há outras soluções.
  
  2)Considere um polinômio de coeficientes 
  inteiros.Sabe-se que p(21)=17,p(32)=-247 e p(37)=33.Prove que se 
  para algum N tivermos p(N)=51.Então N=26.
  
  
  Obrigado pela ajuda.
      
  Eder
  
  


Re: [obm-l] Re:

2002-08-26 Por tôpico Eder



Esse problema apareceu na primeira ou na segunda 
Eureka,se não me engano e o enunciado é assim mesmo.

  - Original Message - 
  From: 
  Augusto 
  César Morgado 
  To: [EMAIL PROTECTED] 
  Sent: Monday, August 26, 2002 7:45 
  PM
  Subject: [obm-l] Re: 
  Eh isso mesmo?A resposta eh nao existe. Claro, se 7/10  p/q  11/15 entao  7/10  np/nq  11/15 e se um q satisfaz, todos os 
  multiplos satisfarao.Eder wrote:
  002301c24d48$fd9241c0$f1f3fea9@Eder" type="cite">



Será que alguém poderia me ajudar neste 
problema:

Se p e q são inteiros positivos tais que 7/10 
 p/q  11/15 ,qual o maior valor que q pode assumir?


Obrigado.


[obm-l] ???

2002-08-19 Por tôpico Eder



Olá,

Aí vão alguns problemas que não estou 
conseguindo resolver:

i)Encontre todas as soluções inteiras de a²-3ab-a+b 
= 0.
ii)Mostre que (8^n )*19+17 é composto para qualquer 
inteiro não-negativo n.

Grato por quaisquer comentários.

Eder


[obm-l] ???

2002-08-13 Por tôpico Eder




Gostaria de ajuda neste 
problema:

Determinar para que valores de n, inteiros e 
positivos ,tem-se 61|(5^n - 4^n). 


Obrigado.



Eder



[obm-l] Re: [obm-l] questão IME

2002-08-10 Por tôpico Eder

Se provarmos que k^5 - k é múltiplo de 10,o problema estará acabado.Vejamos:


i)Pelo Pequeno Teorema de Fermat,temos que k^5=k (mod 5),ou seja,existe c
inteiro tal que k^5-k=c*5.Então k^5-k é múltiplo de 5.
ii)K^5-k=k(k^4-1)=k(k^2-1)(k^2+1)=(k-1)k(k+1)(k^2+1).Observe a presença de
dois interios consecutivos: k e k+1 ou k e k-1 e k.Dentre eles,um é
necessariamente par,o que torna todo o produto múltiplo de 2.

Sendo k^5-k múltiplo de 2 e de 5 ao mesmo tempo,podemos concluir que k^5-k é
múltiplo de 10.Acabado o problema.


Não sei se podemos simplesmente escrever o que está escrito no item i,se a
banca aceitaria.Pensemos outra forma de provar que k^5-k é M5...

Bom,todo inteiro pode ser escrito numa das seguintes formas:
5m,5m+1,5m+2,5m+3 e 5m+4,m inteiro.Daí:

i)Se k=5m,perfeito.
ii)Se k=5m+1,então k-1=5m,perfeito.
iii)Se k=5m+2,então k^2+1= 25m^2+20m+5,que é M5,perfeito.
iv)Se k=5m+3,entãok^2+1=25m^2+30m+10,que é M5,perfeito.
v)Se k=5m+4,então k+1=5m+5,que é M5,perfeito.

Essa seria outra forma.


Eder

- Original Message -
From: rafaelc.l [EMAIL PROTECTED]
To: [EMAIL PROTECTED]
Sent: Saturday, August 10, 2002 5:25 PM
Subject: [obm-l] questão IME



  Por favor, me ajudem a resolver a questão
 abaixo que caiu no IME.


  Provar que para qualquer numero inteiro k,
 os números k e k^5 terminam sempre com o
 mesmo algarismo das unidades.





 obrigado


 __
 AcessoBOL, só R$ 9,90! O menor preço do mercado!
 Assine já! http://www.bol.com.br/acessobol


 =
 Instruções para entrar na lista, sair da lista e usar a lista em
 http://www.mat.puc-rio.br/~nicolau/olimp/obm-l.html
 O administrador desta lista é [EMAIL PROTECTED]
 =

=
Instruções para entrar na lista, sair da lista e usar a lista em
http://www.mat.puc-rio.br/~nicolau/olimp/obm-l.html
O administrador desta lista é [EMAIL PROTECTED]
=



[obm-l] off

2002-08-03 Por tôpico Eder



Olá companheiros de lista,


Estou com um probleminhameio chato.É que toda 
vez que tento fazer o download de um arquivo pdf não sou perguntado se 
querosalvar oarquivo em pasta,o download é iniciado automaticamente 
numa pasta temporária.Algém saberia me dizer como devo proceder para que possa 
escolher a pasta de destino?Agradeço qualquer ajuda e peço desculpas por 
quaisquer incômodos (e se a dúvida for meio básica).É que realemente não vi 
outra saída,senão perguntar a vcs.


Eder


[obm-l] ??

2002-07-07 Por tôpico Eder




Olá,

Gostaria de expor dois problemas que não estou 
conseguindo resolver.Caso alguém queira comentar,agradeço.


1)Resolver a equação 32z^5=(z+1)^5 no campo dos 
complexos.
2)Provar por indução que ( (n+1)/n)^n = 
n (menor ou igual).


Valeu aí por qualquer coisa.

Eder


[obm-l] ????

2002-07-07 Por tôpico Eder



Olá,

Gostaria de expor dois problemas que não estou 
conseguindo resolver.Caso alguém queira comentar,agradeço.


1)Resolver a equação 32z^5=(z+1)^5 no campo dos 
complexos.
2)Provar por indução que ( (n+1)/n)^n = 
n (menor ou igual).


Valeu aí por qualquer coisa.

Eder


Re: [obm-l] ??

2002-07-07 Por tôpico Eder



Esqueci de citar,no segundo problema,que 
n=3.

  - Original Message - 
  From: 
  Marcio 
  To: [EMAIL PROTECTED] 
  Sent: Sunday, July 07, 2002 6:06 PM
  Subject: Re: [obm-l] ??
  
  A primeira equacaoda 2z = w(z+1), onde w eh 
  uma das raizes quintas da unidade. Portanto, as cinco solucoes sao dadas por z 
  = w/(2-w), fazendo w igual a cada uma das raizes exp(2kpi*i/5) (eh soh vc ver 
  que essas 5 raizes servem e sao distintas, logo sao todas as raizes possiveis 
  da eq. polinomial).
  
  No 2o, vc quer mostrar que f(n)=(1+1/n)^n = n 
  sempre, mas f(1)=21... f(2)=25/4  2... 
  
  Marcio
  
- Original Message - 
From: 
Eder 
To: [EMAIL PROTECTED] 
Sent: Sunday, July 07, 2002 12:50 
PM
Subject: [obm-l] ??


Olá,

Gostaria de expor dois problemas que não estou 
conseguindo resolver.Caso alguém queira comentar,agradeço.


1)Resolver a equação 32z^5=(z+1)^5 no campo dos 
complexos.
2)Provar por indução que ( (n+1)/n)^n 
= n (menor ou igual).


Valeu aí por qualquer coisa.

Eder


[obm-l] Re: [obm-l] RES: [obm-l] valor mínimo

2002-05-20 Por tôpico Eder



Valeu Ralph,


Essa expressão surgiu do seguinte problema: 
detrerminar o menor caminho que uma formiguinha pode fazer por sobre a 
superfície de um cubo de aresta 1,de um vértice a outro "diagonalmente 
oposto".

Eu admiti uma trajetória genérica e cheguei a esse 
valor para o caminho.Eu já imaginava que a=1/2,mas queria provar 
algebricamente.

  - Original Message - 
  From: 
  Ralph Teixeira 
  To: '[EMAIL PROTECTED]' 
  Sent: Monday, May 20, 2002 3:55 PM
  Subject: [obm-l] RES: [obm-l] valor 
  mínimo
  
  É 
  possível sim.
  
  1) 
  Via cálculo
  
  Derive a expressão com relação a "a", iguale a zero, dá uma equação 
  meio feia mas sai que a=1/2;
  
   1.1)Cálculo incrementado
  Note que, se f(x)=sqrt(1+(1-x)^2)+sqrt(1+x^2), 
  então f(x)=f(1-x), isto é, o gráfico da função é simétrico com relação à reta 
  x=1/2. Isto sugere fazer y=x-1/2, e então f(y)=sqrt(1+(y-1/2)^2)+sqrt(1+(y+1/2)^2). Os cálculos 
  aqui já são um pouco mais simples... Dá até para fazer a partir daqui sem 
  cálculo, com mágica 
  
  2) 
  Por geometria
  
  ...mas se você quer uma maneira BEM mágica de fazer, 
  pense assim:
  
  f(a)=sqrt(1+(1-a)^2)+sqrt(1+a^2) é a soma das distâncias 
  do ponto (1,a) aos pontos (0,1) e (2,0). Em outras palavras, queremos o ponto 
  P(1,a) na reta x=1 que minimiza as somas das distâncias aos pontos B(0,1) e 
  C(2,0) -- que são fixos e se encontram um de cada lado da reta! Ora, o menor 
  caminho BPC é o segmento de reta que liga B a C! Assim, o mínimo se dá quando 
  B,P e C estão alinhados; note que, então, P será o ponto médio de BC, isto é, 
  a=1/2.
  
  Legal?
  
  Abraço,
   
  Ralph
  
  -Mensagem 
  original-De: Eder 
  [mailto:[EMAIL PROTECTED]]Enviada em: sexta-feira, 17 de 
  maio de 2002 21:34Para: [EMAIL PROTECTED]Assunto: 
  [obm-l] valor mínimoOlá,É possível 
  determinar para que valor de a,tem-se y= sqrt( 1+ (1-a)²) + sqrt(1+ a²) 
  mínimo?
  
  


[obm-l] ??

2002-05-20 Por tôpico Eder



Olá,


Ficarei muito gratoa quem me ajuar com o 
seguinte problema:

"Mostre que para todo m0, sqrt(x)+m=x tem 
exatamente uma raiz."

É do volume 1 da coleção Matemática para o ensino 
médio.


Re: [obm-l] Re: ???

2002-05-01 Por tôpico Eder



Alguns colegas meus acharam 67.Ninguém achou 
65,ainda.Eu considerei que o jardineiro partia do centro da fila de mudas e que 
ia colocando 3 mudas no lado direito,3 no lado esquerdo,...,e cheguei a uma 
distância total de 35 hectômetros.Porém,eu somei a distância da última viagem,o 
que não seria necessário,pela sua resolução.Vamos ver se alguém esclarece 
isso...


- Original Message - 

  From: 
  Alexandre 
  Tessarollo 
  To: [EMAIL PROTECTED] 
  Sent: Wednesday, May 01, 2002 4:26 
  AM
  Subject: [obm-l] Re: ???
   Bem, ele planta a primeira muda, anda 
  5m, planta a 2a, anda +5m, plantaa terceira e volta 10m até a origem. 
  Logo, sendo d[n] a distância percorrida p/plantar o n-ésimo triode 
  árvores, temos d[1]=20m. P/plantar a 4a muda, ele 
  anda 10+5m, depois +5 p/a 5a, +5 p/a sextae volta +25m até a origem. Ou 
  seja, d[2]=50m. Se vc observar, 
  d[n]=2d[n-1]+2*(5+5+5) ou 
  seja:d[n]=2*(d[n-1]+15). Assim, 
  temosd[1]=20d[2]=50d[3]=130d[4]=290d[5]=610d[6]=1250d[7]=2550 
  ...Confesso que não consegui achar nenhum padrão nessa sequencia, a 
  não serqued[n]=3d[n-1]-d[n-2] para 
  n2d[n]=8d[n-2]-3d[n-3] para 
  n3 Então, tentemos de uma forma mais 
  simples:os primeiros 10m ele percorre 2n vezes no trio n de mudas 
  (10*2*n);os 15m seguintes ele percorre 2(n-1) vezes (15*2*(n-1));os 
  15m depois, 2(n-2) vezes (15*2*(n-2));e assim 
  sucessivamente. Lembrando que há 21 trios de mudas, 
  a distância total percorrida 
  seriadeD=10*2*21+15*2*20+15*2*19+...15*2*1D=420+15*2*(20+19+18+...+1)D=6720 
  Vale lembra que, após plantar a última muda o jardineiro NÃO precisavoltar 
  a origem. Portanto, devemos excluir a distância entre a última e a 
  primeira muda, ou seja, precisamosexcluir 100m da conta - se alguém 
  não entedeu pq 100m, imagine 21 MARCOS numa estrada, distantes5m uns dos 
  outros e tente achar a distância total. Assim, a 
  distância percorrida pelo nosso jardineiro do plantio da primeiramuda até 
  o plantio da última muda é de 6620m ou, se vc preferir, 66,2 hectômetros(a 
  abreviação é hm ouHm?) Se você puder colocar a 
  resoluçào daquelas pessoas que acharm 65 serialegal. Obviamente, uma das 
  2 está errada e a outra talvez esteja certa. De qq forma, comentem a 
  minharesposta e vamos ver como se acha 
  65...[]'sAlexandre TessarolloPS: NÃO tenho certeza 
  dessa solução, ainda vou revisá-la com a devida calma.Achei melhor 
  publicá-la logo pq sei q depois de revisá-la num papel não vou ter a 
  necessária paciênciap/digitar... :-)) A propósito., minhas resolução e 
  resposta bateram com a sua? Caso não, publique-asvc tb!Eder 
  wrote: Olá colegas de lista, O seguinte problema,proposto 
  em um vestibular daUNB,está causando uma  controvérsia lá no 
  colégio...Tenho um gabarito dizendo que a respostaé 65,porém eu não 
  consigo chegar a  esse resultado,nem alguns colegas de sala.Se 
  alguém puder resolver,agradeço.No projeto urbanístico de  
  uma cidade ,o paisagista previu a urbanização do canteiro central de 
  umadas avenidas,com o plantio de  63 mudas de Flamboyant,todas 
  dispostas em linha reta e distantes 5m umada outra.No dia do plantio,o 
   caminhão descarregou as mudas no canteiro central,no local onde 
  seriaplantada a primeira muda.Um  jardineiro foi designado para 
  executar o serviço.Para isso,partindo dolugar onde as mudas foram 
   colocadas,ele pegou 3 mudas de cada vez,plantou-as nos locais 
  designados,enfileirando-asuma após a  outra.Calcule ,em 
  hectômetros,a distância total mínima percorrida pelojardineiro após 
  finalizar o 
  trabalho.=Instruções 
  para entrar na lista, sair da lista e usar a lista emhttp://www.mat.puc-rio.br/~nicolau/olimp/obm-l.htmlO 
  administrador desta lista é [EMAIL PROTECTED]=


[obm-l] ???

2002-04-30 Por tôpico Eder



Olá colegas de lista,

O seguinte problema,proposto em um vestibular da 
UNB,está causando uma controvérsia lá no colégio...Tenho um gabarito dizendo que 
a resposta é 65,porém eu não consigo chegar a esse resultado,nem alguns colegas 
de sala.Se alguém puder resolver,agradeço.


No projeto urbanístico de uma cidade ,o 
paisagista previu a urbanização do canteiro central de uma das avenidas,com o 
plantio de 63 mudas de Flamboyant,todas dispostas em linha reta e distantes 5m 
uma da outra.No dia do plantio,o caminhão descarregou as mudas no canteiro 
central,no local onde seria plantada a primeira muda.Um jardineiro foi designado 
para executar o serviço.Para isso,partindo do lugar onde as mudas foram 
colocadas,ele pegou 3 mudas de cada vez,plantou-as nos locais 
designados,enfileirando-as uma após a outra.Calcule,em hectômetros,a 
distância total mínima percorrida pelo jardineiro após finalizar o 
trabalho.






[obm-l] Re: [obm-l] Re: [obm-l] demonstração

2002-04-28 Por tôpico Eder



Obrigado pela solução.

  - Original Message - 
  From: 
  Lucelindo D. 
  Ferreira 
  To: [EMAIL PROTECTED] 
  Sent: Saturday, April 27, 2002 5:23 
  PM
  Subject: [obm-l] Re: [obm-l] 
  demonstração
  
  E aí Eber tudo blz! 
  Tudo começa com a Lei dos Senos 
  observe que senA = senA', senC = sen(a+B), 
  senC' = sen(A-B).Então pela famosa lei dos senos.
  a/senA=b/senB=c/sen(A+B)
  a'/senA = b'/senB=c'/sen(A-B)
  
  aa'/(senA)^2 = bb'/ (senB)^2 = cc'/[(senAcosB)^2 
  - (senBcosA)^2]
  
  bb' = aa'(senB^2)/(senA^2)
  cc' = aa'[(senAcosB)^2 - 
  (senBcosA)^2]/(senA)^2
  bb' +cc' = aa'[(senAcosB)^2+ 
  senB^2(1-cosA^2)]/(senA)^2 = aa'[ (senAcosB)^2 + (senBsenA)^2]/(senA)^2= 
  aa'[senA^2(cosB^2 + senB^2)/(senA)^2= aa'.
   
  See you later
  
- Original Message - 
From: 
Eder 
To: [EMAIL PROTECTED] 
Sent: Monday, April 22, 2002 5:45 
PM
Subject: [obm-l] demonstração

Num tô conseguindo...

"Dados doi triângulos ABC e A'B'C' nos quais 
A+A'=180º e B=B',demonstre que 
aa'=bb'+cc'."

Obrigado por qualquer 
  ajuda.


[obm-l] demonstração

2002-04-24 Por tôpico Eder



Num tô conseguindo...

"Dados doi triângulos ABC e A'B'C' nos quais 
A+A'=180º e B=B',demonstre que 
aa'=bb'+cc'."

Obrigado por qualquer 
ajuda.


[obm-l] AS

2002-03-04 Por tôpico Eder



Olá,


Eu gostaria de pedir licensa para colocar uma 
dúvida que talvez esteja mais relacionada a Física que a Matemática,ou não...É o 
seguinte:

As grandezas da relação abaixo foram medidas sem 
erro até o penúltimo algarismo significativo.Especifique o erro relativo máximo 
e o número de algarismos significativos de cada uma destas 
grandezas:

a)0,0093m
b)0,08275m


O número de AS são 2 e 4 ,respectivamente,se não me 
engano.Agora,como calcular esse erro relativo máximo?

Mais uma vez,peço desculpas a quem não achar 
conveniente esta mensagem.

Eder


[obm-l] Re

2002-02-21 Por tôpico Eder




Valeu pela resolução David e demais 
companheiros de lista.

Eu gostaria de propor mais duas:


1)Seja f:R==R,não identicamente nula,tal 
que

f(x)*f(y)=(1/2)[f(x+y)+f(x-y)] e f(1)=0,para 
todos os números reais x e y.

a)Mostre que f(0)=1,f(2)=-1,f(3)=0 e 
f(4)=1.
b)Mostre que f(x+4)=f(x),para todo x 
real.
c)Existe de fato tal função.

É fácil verificar o item a,mas não consegui o b e o 
c.

2)Seja p(x)=x^3+ax^2+bx+c um polinômio com 
coeficientes inteiros.Suponha que a equação p(x)-0 tem raízes inteiras 
distintas.Mostre que a equação p(x)-1=0 não admite nenhuma raiz 
inteira.

3)Dada uma equação do segundo grau, com 
coeficientes inteiros,mostre que seu discriminante não pode ser igual a 
23.


Essa eu acho que consegui fazer.Como eu não tenho 
muita prática em problemas de olimpíada,vou esboçar minha resolução.Quem vir 
alguma besteira,pode comentar se quiser.

Fiz y=ax²+bx+c (com a,b e c nas condições do 
enunciado)

Observei que todo quadrado perfeito termina em 
0,1,4,5,6 ou 9 e que os múltiplos de 4 terminam em 
0,2,4,6 ou 8.

Daí verifiquei o algarismo das unidades de 
delta=b²-4ac admitindo b² terminando em 0,1,4,5,6 ou 9.

As possibilidades de 3 ser o algarismo das unidades 
de delta apareceram para b² terminando em 1 e b² terminando em 9.

Para b² terminando em 1,temos que b termina em 1 ou 
9.Daí b pertence {+-1,+-9,+-11,+-19,...}.
Esses números são da forma 4k+ -1,k 
inteiro.

(4k+ -1)²=16k²+-8k+1

Como delta =b²-4ac,fiz delta igual a 
23:

16k²+-8k+1-4ac=23 == 2(k²+-2k-ac)=11 == 11 
émúltiplo de 2(absurdo)

Para b² terminando em 5,temos b múltiplo ímpar de 
5.Pondo b=5(2k+1):

delta=25(4k²+4k+1)-4ac

Fazendo delta=23,resulta 25k²+25k-ac=(-1/2).Mas 
sendo k,a e c inteiros isso não pode acontecer.

Delta nunca é igual a 23.

  - Original Message - 
  From: 
  David 
  Daniel Turchick 
  To: [EMAIL PROTECTED] 
  Sent: Thursday, February 21, 2002 12:36 
  AM
  Subject: Re: [obm-l] ???
  
  Eder, eu mandei e-mail respondendo à sua 
  dúvida prá lista, mas por algum motivo ele não chegou, sei lá pq... Aí vai a 
  minha resposta.
  
  
  
  
  Você conhece o Teorema de Cramer? Ele diz que 
  um sistema linear de n equações a n incógnitas tem solução única se, e somente 
  se, o determinante da matriz dos coeficientes for não-nulo.
  
  Sendo os pontos (x_1,y_1), (x_2,y_2) e (x_3,y_3), queremos 
  encontrar a,b,c reais tais que a*(x_i)^2+b*x_1+c=y_i, 
  i=1,2,3. Acabamos então de montar um sistema linear de 3 equações a 3 
  incógnitas, a, b e c. A matriz dos coeficientes é {[(x_1)^2, x_1, 1], 
  [(x_2)^2, x_2, 1], [(x_3)^2, x_3, 1]}, cujo determinante é 
  (x_2-x_1)*(x_3-x_1)*(x_3-x_2) (a matriz é de Vandermonde, então é fácil). Isso 
  só seria zero se tivéssemos coincidência de pelo menos duas abscissas, o que 
  você explicitou não acontecer. Logo, pelo Teorema de Cramer, EXISTEM ÚNICOS 
  a,b,c reais que satisfazem o sistema. Fora isso, o a não é zero, pois se 
  fosse, teríamos a reta bx+c passando pelos três pontos (que você disse serem 
  não-colineares). Logo, existe uma única parábola passando pelos três 
  pontos.
  
  Você pode verificar que esse argumento (até a parte do "a 
  não é zero, pois...") continua valendo para um caso mais geral: por n pontos 
  de RxR com abscissas distintas 2 a 2, passa no máximo uma função polinomial de 
  grau n-1.
  
  David
  
-Mensagem original-De: 
Eder [EMAIL PROTECTED]Para: 
[EMAIL PROTECTED] [EMAIL PROTECTED]Data: 
Quarta-feira, 20 de Fevereiro de 2002 16:38Assunto: [obm-l] 
???
Olá,

Será que alguém poderia ajudar nesta 
questão:

"Considere três pontos no plano cartesiano,não 
colineares e com abcissas distintas duas a duas.Qual o número de funções 
quadráticas que podem ser encontradas de maneira que esses pontos pertençam 
aos seus gráficos?"

Essa questão foi do vestibular de uma 
universidade não lá muito conceituada,mas eu ainda não matei a 
charada...


[obm-l] ???

2002-02-20 Por tôpico Eder



Olá,

Será que alguém poderia ajudar nesta 
questão:

"Considere três pontos no plano cartesiano,não 
colineares e com abcissas distintas duas a duas.Qual o número de funções 
quadráticas que podem ser encontradas de maneira que esses pontos pertençam aos 
seus gráficos?"

Essa questão foi do vestibular de uma universidade 
não lá muito conceituada,mas eu ainda não matei a 
charada...


Re: iezzi

2002-01-06 Por tôpico Eder



2940x= m^3== 
(2^2)*3*5*(7^2)x=m^3

Completamos os cubos fazendo x=2*(3^2)*7*(5^2) 
== x= 3150 (valor mínimo).Assim podemos extrair a raiz cúbica em ambos 
lados,"m" resultando inteiro.

  - Original Message - 
  From: 
  gabriel 
  guedes 
  To: [EMAIL PROTECTED] 
  Sent: Sunday, January 06, 2002 1:45 
  PM
  Subject: iezzi
  
  Ola amigos da lista,
  estou com difilculdades neste problema 
  :
  
  Determine o menor numero 
  inteiro positivo x para q 2940x= m^3 em q m é um inteiro 
  positivo.
  
  Abraços Gabriel.
  


triângulo

2002-01-06 Por tôpico Eder



No triângulo ABC C=3A (ângulos),a=27 e 
c=48.Quanto mede b?

Até agora não resolvi essa...

Outra:

Se (a^b)=(b^a) e b=(9^a),qual o valor de 
a?

Conto com a ajuda dos colegas de 
lista.


  1   2   >